You have already completed the quiz before. Hence you can not start it again.
Quiz is loading...
You must sign in or sign up to start the quiz.
You have to finish following quiz, to start this quiz:
Results
0 of 10 questions answered correctly
Your time:
Time has elapsed
You have reached 0 of 0 points, (0)
Average score
Your score
Categories
Not categorized0%
Hematology0%
1
2
3
4
5
6
7
8
9
10
Answered
Review
Question 1 of 10
1. Question
A 32-year-old woman, who is at 20 weeks gestational age, presents to the ED after a seizure. Her vital signs are BP 115/70, HR 105, RR 16, T 38.5°C, and pulse oximetry 98% on room air. On exam, you note some confusion, but otherwise there are no focal deficits. Lab results reveal a hemoglobin of 7 g/dL and platelets of 12,000/µL. A peripheral blood smear reveals schistocytes. Which of the following is the most appropriate treatment for her condition?
Correct
The patient has thrombotic thrombocytopenic purpura (TTP). The classic pentad of TTP includes CNS abnormalities, renal pathology, fever, microangiopathic hemolytic anemia, and thrombocytopenia. However, diagnostic criteria have recently been simplified to include all adults with microangiopathic or microvascular hemolytic anemia and thrombocytopenia with no other explanation for these findings. TTP shares many clinical and laboratory features of HELLP syndrome. HELLP syndrome is less common before 24 weeks gestation. The derangements in hemoglobin and platelet levels are more severe in TTP. The mainstay of treatment for TTP is plasmapheresis (plasma exchange), which can achieve remission of disease in 80% of patients. If plasmapheresis cannot be immediately performed, fresh frozen plasma (FFP) should be administered until apheresis can be performed.
Incorrect
The patient has thrombotic thrombocytopenic purpura (TTP). The classic pentad of TTP includes CNS abnormalities, renal pathology, fever, microangiopathic hemolytic anemia, and thrombocytopenia. However, diagnostic criteria have recently been simplified to include all adults with microangiopathic or microvascular hemolytic anemia and thrombocytopenia with no other explanation for these findings. TTP shares many clinical and laboratory features of HELLP syndrome. HELLP syndrome is less common before 24 weeks gestation. The derangements in hemoglobin and platelet levels are more severe in TTP. The mainstay of treatment for TTP is plasmapheresis (plasma exchange), which can achieve remission of disease in 80% of patients. If plasmapheresis cannot be immediately performed, fresh frozen plasma (FFP) should be administered until apheresis can be performed.
Question 2 of 10
2. Question
Which of the following conditions include indications for use of DDAVP in acute bleeding?
Correct
DDAVP stimulates the release of VWF, Factor VIII, and tissue factor. It may be used in bleeding episodes with mild Hemophilia A and von Willebrand disease. It does not affect Factor IX and so would not help in Hemophilia B. DDAVP does have beneficial effects on platelet function but would not help in the setting of thrombocytopenia.
Incorrect
DDAVP stimulates the release of VWF, Factor VIII, and tissue factor. It may be used in bleeding episodes with mild Hemophilia A and von Willebrand disease. It does not affect Factor IX and so would not help in Hemophilia B. DDAVP does have beneficial effects on platelet function but would not help in the setting of thrombocytopenia.
Question 3 of 10
3. Question
The value of obtaining a D-dimer in a patient suspected of having DIC rests in its HIGH:
Correct
D-dimer has an NPV of 99% in the diagnosis of DIC. If this test is negative, the clinician can be nearly certain the patient does not have DIC.
Incorrect
D-dimer has an NPV of 99% in the diagnosis of DIC. If this test is negative, the clinician can be nearly certain the patient does not have DIC.
Question 4 of 10
4. Question
A 45-year-old male with a history of Hodgkin’s lymphoma presents with a chief complaint of nausea/vomiting and muscle spasms. Physical exam reveals hyper-reflexia, and a positive Chvostek’s sign. The basic metabolic panel is significant for an elevated creatinine, and hyperkalemia. What lab test will most likely reveal the diagnosis?
Correct
This patient most likely has tumor lysis syndrome, characterized by hyperkalemia, hyperphosphatemia, hyperuricemia, and hypocalcemia. The tetany (chvostek’s sign of tapping on facial nerve causing muscle twitching) and hyper-reflexia are likely secondary to the hypocalcemia. A uric acid level is the most important test to aid in this diagnosis. Treatment is generally supportive with fluids and correction of electrolyte abnormalities. While hypoparathyroidism can cause low calcium, it will not cause the renal insufficiency seen in tumor lysis syndrome. Neither a CBC nor a peripheral smear are likely to aid in this diagnosis.
Incorrect
This patient most likely has tumor lysis syndrome, characterized by hyperkalemia, hyperphosphatemia, hyperuricemia, and hypocalcemia. The tetany (chvostek’s sign of tapping on facial nerve causing muscle twitching) and hyper-reflexia are likely secondary to the hypocalcemia. A uric acid level is the most important test to aid in this diagnosis. Treatment is generally supportive with fluids and correction of electrolyte abnormalities. While hypoparathyroidism can cause low calcium, it will not cause the renal insufficiency seen in tumor lysis syndrome. Neither a CBC nor a peripheral smear are likely to aid in this diagnosis.
Question 5 of 10
5. Question
A 2 year old child is brought in with a past medical history significant for fever and bloody diarrhea. Based on laboratory testing, a presumptive diagnosis of Hemolytic Uremic Syndrome is made. What is the most significant cause of potential morbidity and mortality in this patient?
Correct
The patient in this question has hemolytic uremic syndrome, of which the acute renal failure is the leading cause of morbidity and mortality. If anuric, these patients require plasma exchange therapy, dialysis, and/or monoclonal antibody treatment.
Incorrect
The patient in this question has hemolytic uremic syndrome, of which the acute renal failure is the leading cause of morbidity and mortality. If anuric, these patients require plasma exchange therapy, dialysis, and/or monoclonal antibody treatment.
Question 6 of 10
6. Question
A 68-year-old male with a history of chronic myelogenous leukemia presents to the emergency department with severe shortness of breath. He is diaphoretic and hypoxic to 86% on room air. CBC reveals a white blood cell count of greater than 200,000, with >90% blasts. Hematology has been paged. Which of the following is the definitive next step in management for this patient?
Correct
This patient is most likely suffering from leukostasis secondary to blast crisis. The most important treatment in this case is immediate induction chemotherapy. If induction chemotherapy is unavailable, leukaphoresis and hydroxyurea may be used as temporizing measures but the definitive treatment is induction chemotherapy. IV rasburicase is the treatment for severe tumor lysis syndrome.
Incorrect
This patient is most likely suffering from leukostasis secondary to blast crisis. The most important treatment in this case is immediate induction chemotherapy. If induction chemotherapy is unavailable, leukaphoresis and hydroxyurea may be used as temporizing measures but the definitive treatment is induction chemotherapy. IV rasburicase is the treatment for severe tumor lysis syndrome.
Question 7 of 10
7. Question
A 60-year-old female presents with a chief complaint of progressive fatigue and shortness of breath for 2 months. Electrolytes are grossly normal, and a CBC is significant for a WBC count of 55,000, hemoglobin of 6 g/dL, hematocrit of 18%, and a platelet count of 20,000. Physical exam reveals petechiae, but no lymphadenopathy or hepatosplenomegaly. Examination of the oropharynx is shown below. What is the most likely diagnosis?
Correct
This patient most likely has AML, which can present with the above gingival findings due to infiltration by tumor cells. Of note, the CBC can vary in patients with AML. Normocytic anemia and thrombocytopenia are common, however the white blood cell count may be low, normal, or elevated. Lymphadenopathy is rare in AML, in contrast to ALL in which it is a common presenting symptom. Definitive diagnosis of AML is by bone marrow biopsy.
Incorrect
This patient most likely has AML, which can present with the above gingival findings due to infiltration by tumor cells. Of note, the CBC can vary in patients with AML. Normocytic anemia and thrombocytopenia are common, however the white blood cell count may be low, normal, or elevated. Lymphadenopathy is rare in AML, in contrast to ALL in which it is a common presenting symptom. Definitive diagnosis of AML is by bone marrow biopsy.
Question 8 of 10
8. Question
A 57-year-old woman with multiple myeloma presents with myalgias, abdominal pain, generalized weakness, and confusion. Laboratory testing demonstrates a calcium of 15.5 mg/dL. Aggressive hydration with normal saline is initiated. Which of the following medications is also indicated?
Correct
Hypercalcemia is generally a product of another underlying disorder and not a primary process in itself. Causes are grouped into four categories: malignancy (primary hematologic, metastases to the bone, or parathyroid producing tumor), hyperparathyroidism, increased intake (milk-alkali syndrome, vitamin D or A toxicity), and increased bone breakdown (immobilization, Paget disease). Clinically, patients experience lethargy, weakness, myalgias, constipation, and anorexia. The clinical presentation is often remembered by the mnemonic “bones, stones, groans and psychiatric overtones.” The first step in treatment is aggressive hydration. The addition of intravenous bisphosphonates (e.g. zoledronic acid) inhibits calcium release from bone. It is usually reserved for hypercalcemia associated with malignancy.
Incorrect
Hypercalcemia is generally a product of another underlying disorder and not a primary process in itself. Causes are grouped into four categories: malignancy (primary hematologic, metastases to the bone, or parathyroid producing tumor), hyperparathyroidism, increased intake (milk-alkali syndrome, vitamin D or A toxicity), and increased bone breakdown (immobilization, Paget disease). Clinically, patients experience lethargy, weakness, myalgias, constipation, and anorexia. The clinical presentation is often remembered by the mnemonic “bones, stones, groans and psychiatric overtones.” The first step in treatment is aggressive hydration. The addition of intravenous bisphosphonates (e.g. zoledronic acid) inhibits calcium release from bone. It is usually reserved for hypercalcemia associated with malignancy.
Question 9 of 10
9. Question
A 40-year-old woman presents to the ED with confusion. Blood pressure is 130/80, HR 90, RR 20, 98% oxygen saturation on room air, and T 98.5 °F. On physical exam, she is alert and oriented to person and time. Scattered petechiae are also noted. Her complete blood count shows white blood cell count 10 x 109/L, hemoglobin of 7.5 g/dL, and platelet 15 x 109/L. Head CT was ordered and revealed no intracranial hemorrhage. Which of the following is the next best step in management?
Correct
Plasma exchange is the treatment for patients presenting with thrombotic thrombocytopenic purpura (TTP). Patients with this diagnosis have high mortality and should be treated as soon as possible. In a patient with altered mental status, severe thrombocytopenia, and neurologic symptoms, thrombotic thrombocytopenic purpura should be suspected. Patients commonly present with weakness, gastrointestinal symptoms, or neurologic symptoms. Neurologic manifestations may include transient neurologic deficits, confusion, coma, and headache. The classic presentation of fever, renal failure, anemia, thrombocytopenia, and severe neurologic symptoms only occurs in a small percentage of patients. The pathophysiology of TTP involves abnormal platelet aggregation secondary to an abnormality in ADAMTS13, which leads to microangiopathic hemolytic anemia, characterized by schistocytes on blood smear, thrombocytopenia, and end-organ damage.
IV immunoglobulin (A) is the treatment for patients presenting with immune thrombocytopenia (ITP). Patients with ITP will have thrombocytopenia without any clear etiology and therefore should be a diagnosis of exclusion. Patients with ITP will not have evidence of microangiopathic hemolytic anemia or signs of end-organ damage. A lumbar puncture (B) is indicated in a patient suspected to have a CNS infection. In a patient with a normal white blood cell count and no fever, a CNS infection is less likely. Also, the chances of an epidural hematoma would be higher in a patient who has a platelet count of 15 x 109/L. The risks of a lumbar puncture outweigh the benefits in a patient with severe thrombocytopenia and low clinical suspicion for infection. A platelet transfusion (D) is unnecessary in a patient with significant thrombocytopenia unless there is evidence of active bleeding or a procedure is indicated and there is a risk of significant bleeding during the intervention. There is also a potential risk with platelet administration in patients with TTP, since platelet aggregation may worsen as the result of transfusion.
Incorrect
Plasma exchange is the treatment for patients presenting with thrombotic thrombocytopenic purpura (TTP). Patients with this diagnosis have high mortality and should be treated as soon as possible. In a patient with altered mental status, severe thrombocytopenia, and neurologic symptoms, thrombotic thrombocytopenic purpura should be suspected. Patients commonly present with weakness, gastrointestinal symptoms, or neurologic symptoms. Neurologic manifestations may include transient neurologic deficits, confusion, coma, and headache. The classic presentation of fever, renal failure, anemia, thrombocytopenia, and severe neurologic symptoms only occurs in a small percentage of patients. The pathophysiology of TTP involves abnormal platelet aggregation secondary to an abnormality in ADAMTS13, which leads to microangiopathic hemolytic anemia, characterized by schistocytes on blood smear, thrombocytopenia, and end-organ damage.
IV immunoglobulin (A) is the treatment for patients presenting with immune thrombocytopenia (ITP). Patients with ITP will have thrombocytopenia without any clear etiology and therefore should be a diagnosis of exclusion. Patients with ITP will not have evidence of microangiopathic hemolytic anemia or signs of end-organ damage. A lumbar puncture (B) is indicated in a patient suspected to have a CNS infection. In a patient with a normal white blood cell count and no fever, a CNS infection is less likely. Also, the chances of an epidural hematoma would be higher in a patient who has a platelet count of 15 x 109/L. The risks of a lumbar puncture outweigh the benefits in a patient with severe thrombocytopenia and low clinical suspicion for infection. A platelet transfusion (D) is unnecessary in a patient with significant thrombocytopenia unless there is evidence of active bleeding or a procedure is indicated and there is a risk of significant bleeding during the intervention. There is also a potential risk with platelet administration in patients with TTP, since platelet aggregation may worsen as the result of transfusion.
Question 10 of 10
10. Question
Which of the following well appearing patients with a temperature of greater than 38.4oC requires prompt evaluation and intravenous antibiotic therapy within one hour of arrival to the emergency department?
Correct
Severe neutropenia is defined as having an absolute neutrophil count less than 500 cells/microL which increases the risk of spontaneous serious bacterial infection. Neutropenia can be inherited or more usually acquired while on chemotherapy. Viral infections can cause transient neutropenia and if discovered incidentally further workup may be indicated. Patients with known severe neutropenia such as a patient with a neutrophil count less than 500 cells/microL in the last week who presents to the emergency department with a fever should be immediately evaluated and started on broad–spectrum antibiotics, even if well appearing. A delay of more than 60 minutes in antibiotic administration to patients with severe neutropenia is associated with an increase in morbidity, mortality, and length of stay.
Incorrect
Severe neutropenia is defined as having an absolute neutrophil count less than 500 cells/microL which increases the risk of spontaneous serious bacterial infection. Neutropenia can be inherited or more usually acquired while on chemotherapy. Viral infections can cause transient neutropenia and if discovered incidentally further workup may be indicated. Patients with known severe neutropenia such as a patient with a neutrophil count less than 500 cells/microL in the last week who presents to the emergency department with a fever should be immediately evaluated and started on broad–spectrum antibiotics, even if well appearing. A delay of more than 60 minutes in antibiotic administration to patients with severe neutropenia is associated with an increase in morbidity, mortality, and length of stay.
NOTE:Next week (March 21st) is Critical Care Conference. The following week will be Hematology. For the conference, study up on resus ultrasound (RUSH exam, FAST, cardiac, etc.), Sepsis (just went over this!), and pediatric resus (Peds EM Playbook), as this is all usually covered.
Time limit: 0
Quiz-summary
0 of 10 questions completed
Questions:
1
2
3
4
5
6
7
8
9
10
Information
You have 10 minutes to answer 10 questions
You have already completed the quiz before. Hence you can not start it again.
Quiz is loading...
You must sign in or sign up to start the quiz.
You have to finish following quiz, to start this quiz:
Results
0 of 10 questions answered correctly
Your time:
Time has elapsed
You have reached 0 of 0 points, (0)
Average score
Your score
Categories
Not categorized0%
Hematology0%
Immunology0%
1
2
3
4
5
6
7
8
9
10
Answered
Review
Question 1 of 10
1. Question
A 23-year-old man with sickle cell disease presents with chest pain, cough, and fever. His vitals are HR 132, RR 28, BP 110/65, and T 101.1°F. His chest X-ray shows a right lower lobe infiltrate. Hemoglobin is 8 g/dL. Which of the following is the first line management that is most likely indicated?
Correct
This patient with sickle cell disease (SCD) presents with acute chest syndrome requiring broad-spectrum antibioticsand intensive care unit (ICU) admission. SCD is a genetically determined disease due to an abnormal allele for hemoglobin beta chains. The end result of this abnormality is a sickled cell that is less deformable and can cause increased viscosity and sludging of the blood. Additionally, cells are sequestered in the spleen and the liver leading to destruction. Patients experience chronic hemolysis, vaso-occlusive events, thrombosis and ultimately, end organ injury. Splenic autoinfarction occurs early in life and makes patients more susceptible to encapsulated organisms such asStreptococcus pneumoniae, Haemophilus influenzae and Neisseria meningitides. Acute chest syndrome is the leading cause of death in SCD and is defined as fever, chest pain, and the presence of new pulmonary infiltrates. Management consists of supportive care, supplemental oxygen if needed, broad spectrum antibiotics, andadmission to the ICU.
Incorrect
This patient with sickle cell disease (SCD) presents with acute chest syndrome requiring broad-spectrum antibioticsand intensive care unit (ICU) admission. SCD is a genetically determined disease due to an abnormal allele for hemoglobin beta chains. The end result of this abnormality is a sickled cell that is less deformable and can cause increased viscosity and sludging of the blood. Additionally, cells are sequestered in the spleen and the liver leading to destruction. Patients experience chronic hemolysis, vaso-occlusive events, thrombosis and ultimately, end organ injury. Splenic autoinfarction occurs early in life and makes patients more susceptible to encapsulated organisms such asStreptococcus pneumoniae, Haemophilus influenzae and Neisseria meningitides. Acute chest syndrome is the leading cause of death in SCD and is defined as fever, chest pain, and the presence of new pulmonary infiltrates. Management consists of supportive care, supplemental oxygen if needed, broad spectrum antibiotics, andadmission to the ICU.
Question 2 of 10
2. Question
A 35 year old male with history of sickle cell anemia presents to the emergency department with severe total body pain. Vital signs are notable for HR of 115. On exam, the patient is diffusely tender to palpation in the chest and bilateral lower extremities. Which of the following is the most appropriate management of this patient?
Correct
The patient in the question stem is suffering from vasoocculsive crisis, or pain crisis of the bones. In adults, this is the most common in the chest, long bones, and back. The mainstay of treatment is narcotics and IV fluids.
Incorrect
The patient in the question stem is suffering from vasoocculsive crisis, or pain crisis of the bones. In adults, this is the most common in the chest, long bones, and back. The mainstay of treatment is narcotics and IV fluids.
Question 3 of 10
3. Question
A 9-year-old African American boy with sickle cell disease and multiple previous admissions for pain crises, is brought in by parents for weakness that started 30 minutes ago. He has not had any recent illnesses. On exam, he has flaccid paralysis on right upper and lower extremities, with decreased light-touch sensation. Bedside hemoglobin is 9.0g/dL. Which of the following is the definitive management for this patient?
Correct
Acute cerebrovascular accident is a complication in sickle cell patients due to intravascular sickling and vasoocclusion. It typically manifests as ischemic changes of large intracranial arteries. However, in older patients, hemorrhagic changes can be seen as arterial walls have been weakened from prior infarcts. The ideal treatment is exchange transfusion; it is the most effective method to reduce the percentage of sickled hemoglobin and to reduce intravascular sickling and vasoocclusion. Exchange transfusion also helps minimize the risk of transfusion-associated circulatory overload and associated pulmonary edema, in addition to reduced risk of stroke recurrence. Unfortunately, there are no published evidence regarding the safety and efficacy of thrombolytics in sickle cell patients with acute ischemic strokes. If exchange transfusion is available, this option should be pursued first.
Incorrect
Acute cerebrovascular accident is a complication in sickle cell patients due to intravascular sickling and vasoocclusion. It typically manifests as ischemic changes of large intracranial arteries. However, in older patients, hemorrhagic changes can be seen as arterial walls have been weakened from prior infarcts. The ideal treatment is exchange transfusion; it is the most effective method to reduce the percentage of sickled hemoglobin and to reduce intravascular sickling and vasoocclusion. Exchange transfusion also helps minimize the risk of transfusion-associated circulatory overload and associated pulmonary edema, in addition to reduced risk of stroke recurrence. Unfortunately, there are no published evidence regarding the safety and efficacy of thrombolytics in sickle cell patients with acute ischemic strokes. If exchange transfusion is available, this option should be pursued first.
Question 4 of 10
4. Question
A 19-year old male with a history of sickle cell disease presents with bilateral proximal arm and leg pain with low back pain for one day. He also endorses rhinorrhea. He has been compliant with his folate and hydroxyurea, but ran out of his prescription pain medications. Vital signs are: BP 143/82, P 99, RR 14, O2Sat 99% room air, T 99.0F (37.2C). Physical exam is unremarkable, with clear lungs, benign abdomen, and moist mucous membranes. His extremities and back are nontender with no overlying skin changes. Neurologic exam is benign. Bedside hemoglobin is 8.8 g/dL. Medical records indicate that his baseline hemoglobin is 9.0 g/dL. Which of the following is the next best step in managing this patient?
Correct
This patient is likely suffering a vasoocclusive pain episode, triggered by a likely viral upper respiratory infection. The provided information does not suggest any life-threatening complications (acute stroke, acute chest syndrome, acute bacterial infection, aplastic crisis, symptomatic anemia) that would require further blood tests or imaging at this time.
For patients who are hypovolemic, IV normal saline resuscitation would be indicated. However, there is no indication in this patient that he is dehydrated. If pain is not adequately controlled with initial PO/IM, then IV pain medication may be attempted.
Incorrect
This patient is likely suffering a vasoocclusive pain episode, triggered by a likely viral upper respiratory infection. The provided information does not suggest any life-threatening complications (acute stroke, acute chest syndrome, acute bacterial infection, aplastic crisis, symptomatic anemia) that would require further blood tests or imaging at this time.
For patients who are hypovolemic, IV normal saline resuscitation would be indicated. However, there is no indication in this patient that he is dehydrated. If pain is not adequately controlled with initial PO/IM, then IV pain medication may be attempted.
Question 5 of 10
5. Question
A 21-year-old man with a history of sickle cell disease, presents with generalized body pain, chest pain, coughing, and shortness of breath for 2 days. Vital signs are: BP 109/73, P 132, T 103.3F (39.6C), RR 23, O2Sat 88% room air. Your physical examination notes diffuse tenderness to palpation across the chest and extremities, but no rashes. You note crackles to the right mid-lung zone. Chest xray is shown. Intravenous fluid resuscitation, pain medication, and oxygen are administered, with oxygen saturation improving to 97% on 10L facemask. Bedside hemoglobin is 8.0g/dL. He is able to tolerate oral medications and food. Which of the following is the next best step in management of this patient?
Correct
Acute chest syndrome is the leading cause of death in adult sickle cell patients. It is a syndrome that includes pneumonia, pulmonary infarction, pulmonary embolism, bone marrow infarction, and fat emboli. To make the diagnosis, patient must have consolidation or infiltrate on CXR, and at least 1 of the following symptoms: fever >38.5°C, chest pain, tachypnea, cough, wheezing, or PaO2 <60mmHg. Treatment includes pain control, fluid hydration, antibiotics to cover for both typical and atypical organisms, and oxygen. Bronchodilators may be used, especially if patients already have underlying reactive airway diseases. Glucocorticoids should be avoided as studies have shown rebound vasoocclusionand pain crises following the cessation of steroids.
Although there is limited evidence regarding transfusion therapy in acute chest syndrome, transfusion is considered a mainstay therapy for acute chest syndrome. Some guidelines suggest simple blood transfusion in mild-moderate acute chest syndrome cases, while recommending exchange transfusion in moderate-severe cases.
Mild acute chest syndrome criteria include: O2Sat >90% on room air; segmental/lobar infiltrates involving no more than 1 lobe on CXR; and response to simple transfusion of no more than 2units RBC.
Moderate acute chest syndrome criteria include: O2Sat at least 85% on room air; segmental/lobar infiltrates involving no more than 2 lobes on CXR; and response to transfusion of 3 or more units RBC.
Severe acute chest syndrome criteria include: respiratory failure requiring mechanical ventilation; O2Sat <85% on room air; segmental/lobar infiltrates involving at least 3 lobes on CXR.
Very severe acute chest syndrome criteria include: presence of ARDS or life-threatening lung failure.
Given the initial transcutaneous oxygen saturation 88% on room air, this patient is classified as moderate acute chest syndrome. Hematology should be consulted for transfusion therapy.
Incorrect
Acute chest syndrome is the leading cause of death in adult sickle cell patients. It is a syndrome that includes pneumonia, pulmonary infarction, pulmonary embolism, bone marrow infarction, and fat emboli. To make the diagnosis, patient must have consolidation or infiltrate on CXR, and at least 1 of the following symptoms: fever >38.5°C, chest pain, tachypnea, cough, wheezing, or PaO2 <60mmHg. Treatment includes pain control, fluid hydration, antibiotics to cover for both typical and atypical organisms, and oxygen. Bronchodilators may be used, especially if patients already have underlying reactive airway diseases. Glucocorticoids should be avoided as studies have shown rebound vasoocclusionand pain crises following the cessation of steroids.
Although there is limited evidence regarding transfusion therapy in acute chest syndrome, transfusion is considered a mainstay therapy for acute chest syndrome. Some guidelines suggest simple blood transfusion in mild-moderate acute chest syndrome cases, while recommending exchange transfusion in moderate-severe cases.
Mild acute chest syndrome criteria include: O2Sat >90% on room air; segmental/lobar infiltrates involving no more than 1 lobe on CXR; and response to simple transfusion of no more than 2units RBC.
Moderate acute chest syndrome criteria include: O2Sat at least 85% on room air; segmental/lobar infiltrates involving no more than 2 lobes on CXR; and response to transfusion of 3 or more units RBC.
Severe acute chest syndrome criteria include: respiratory failure requiring mechanical ventilation; O2Sat <85% on room air; segmental/lobar infiltrates involving at least 3 lobes on CXR.
Very severe acute chest syndrome criteria include: presence of ARDS or life-threatening lung failure.
Given the initial transcutaneous oxygen saturation 88% on room air, this patient is classified as moderate acute chest syndrome. Hematology should be consulted for transfusion therapy.
Question 6 of 10
6. Question
A one year old patient is brought into the emergency department by family for swollen and painful hands and feet. Vital signs are within normal limits, and patient is afebrile. The mother denies history of trauma. On exam, the dorsum of the hands and feet are noted to be diffusely swollen and tender, particularly along the digits. The mother notes that there is a strong family history of sickle cell disease. Which of the following is the most likely diagnosis?
Correct
Acute dactylitis is the most common presenting symptom for patients with sickle cell disease. It most common occurs in patients 6-18 months of age, and represents bone infarction. It is not an infection.
Incorrect
Acute dactylitis is the most common presenting symptom for patients with sickle cell disease. It most common occurs in patients 6-18 months of age, and represents bone infarction. It is not an infection.
Question 7 of 10
7. Question
The genotype of SCD associated with the high- est risk for avascular necrosis is:
Correct
The genotype HbSC is associated with a higher level of total hemoglobin and a less-severe course; however, these patients are at greater risk for avascular joint necrosis. In patients with HbS beta thalassemia, the severity of disease is determined by the severity of the thalassemia mutation. Patients with mild HbS beta+ thalassemia will have near-normal lives, whereas the natural history of HbS beta0 thalassemia is similar to HbSS.
Incorrect
The genotype HbSC is associated with a higher level of total hemoglobin and a less-severe course; however, these patients are at greater risk for avascular joint necrosis. In patients with HbS beta thalassemia, the severity of disease is determined by the severity of the thalassemia mutation. Patients with mild HbS beta+ thalassemia will have near-normal lives, whereas the natural history of HbS beta0 thalassemia is similar to HbSS.
Question 8 of 10
8. Question
In patients admitted for VOC, which treatment is indicated to prevent the development of acute chest syndrome?
Correct
A randomized trial of 29 patients with VOC found that the addition of incentive spirometry during hospitalization for VOC was associated with a 37% decrease
in the incidence of pulmonary in infiltrates and atelectasis. Incentive spirometry should be included in the treatment protocol for all SCD patients admitted for VOC. Acute chest syndrome typically develops during inpatient admissions for VOC and pulmonary infiltrate is a clinical finding required for the diagnosis of acute chest syndrome. This low-risk intervention decreases the incidence of pulmonary infiltrates. In vitro and in vivo studies have shown that lowering of serum osmolality with hypotonic fluid can reduce erythrocyte sickling. Excess IV fluids have been anecdotally associated with the development of atelectasis. A prospective observational cohort of 3751 patients with SCD identified atelectasis as a risk factor for the development of acute chest syndrome.
Incorrect
A randomized trial of 29 patients with VOC found that the addition of incentive spirometry during hospitalization for VOC was associated with a 37% decrease
in the incidence of pulmonary in infiltrates and atelectasis. Incentive spirometry should be included in the treatment protocol for all SCD patients admitted for VOC. Acute chest syndrome typically develops during inpatient admissions for VOC and pulmonary infiltrate is a clinical finding required for the diagnosis of acute chest syndrome. This low-risk intervention decreases the incidence of pulmonary infiltrates. In vitro and in vivo studies have shown that lowering of serum osmolality with hypotonic fluid can reduce erythrocyte sickling. Excess IV fluids have been anecdotally associated with the development of atelectasis. A prospective observational cohort of 3751 patients with SCD identified atelectasis as a risk factor for the development of acute chest syndrome.
Question 9 of 10
9. Question
A 28-year-old man presents with a one-day history of rectal bleeding. In the ED, he is hypotensive, thrombocytopenic, and is found to be passing melena. He receives a transfusion of platelets and packed red blood cells as part of his resuscitation. Twenty minutes after the start of his platelet transfusion, his BP is 90 mm Hg systolic, he becomes dyspneic, and his oxygen saturation drops from 99% on room air to 91% on 2L of oxygen supplementation. On exam, you note rales at the lung apices and that he is using accessory muscles to breathe. His chest radiograph shows diffuse interstitial infiltrates. What is the most likely cause of this complication?
Correct
This patient is most likely suffering from transfusion-related acute lung injury (TRALI), one of the leading causes of transfusion-related mortality. It is most closely associated with platelet and fresh frozen plasma transfusions, though cases have been reported with packed red blood cells since there is some residual plasma in the packed cells. Symptoms begin abruptly during transfusion or within six hours and resemble adult respiratory distress syndrome with noncardiogenic pulmonary edema, dyspnea, hypoxemia, and bilateral infiltrates on chest radiograph. It is thought to be caused by granulocyte recruitment and degranulation. As with all transfusion-associated complications, the transfusion should be stopped immediately and supportive care instituted. Most cases resolve spontaneously.
ABO incompatibility (A) causes intravascular hemolysis of transfused red blood cells, producing hemoglobinemia and hemoglobinuria. The onset is immediate. Symptoms include fever, chills, headache, nausea, vomiting, a sensation of chest restriction, and severe joint pain. ABO incompatibility is most often due to human error. An allergic transfusion reaction (B) can vary in severity from simple pruritus with urticaria to anaphylaxis with wheezing and bronchospasm. This patient’s respiratory symptoms are not bronchospastic in nature. Although TRALI can be confused with acute transfusion-associated circulatory overload (TACO) (C), TRALI is associated with hypotension, whereas TACO is associated with a rapid rise in blood pressure.
Incorrect
This patient is most likely suffering from transfusion-related acute lung injury (TRALI), one of the leading causes of transfusion-related mortality. It is most closely associated with platelet and fresh frozen plasma transfusions, though cases have been reported with packed red blood cells since there is some residual plasma in the packed cells. Symptoms begin abruptly during transfusion or within six hours and resemble adult respiratory distress syndrome with noncardiogenic pulmonary edema, dyspnea, hypoxemia, and bilateral infiltrates on chest radiograph. It is thought to be caused by granulocyte recruitment and degranulation. As with all transfusion-associated complications, the transfusion should be stopped immediately and supportive care instituted. Most cases resolve spontaneously.
ABO incompatibility (A) causes intravascular hemolysis of transfused red blood cells, producing hemoglobinemia and hemoglobinuria. The onset is immediate. Symptoms include fever, chills, headache, nausea, vomiting, a sensation of chest restriction, and severe joint pain. ABO incompatibility is most often due to human error. An allergic transfusion reaction (B) can vary in severity from simple pruritus with urticaria to anaphylaxis with wheezing and bronchospasm. This patient’s respiratory symptoms are not bronchospastic in nature. Although TRALI can be confused with acute transfusion-associated circulatory overload (TACO) (C), TRALI is associated with hypotension, whereas TACO is associated with a rapid rise in blood pressure.
Question 10 of 10
10. Question
A 39-year-old man presents to the emergency department for fatigue and a rash. He was treated with chloramphenicol for a recent zoonotic infection. His vital signs are T 99°F, P 105 beats/minute, RR 12 breaths/minute, BP 139/85 mm Hg, and oxygen saturation 100% on room air. He has petechiae on his trunk and extremities. Laboratory analysis reveals a hemoglobin of 6.8 g/dL, white blood cell count 0.9 cells/mcL, platelets 20 000 cells/mcL, and reticulocyte count 0.2%. What is the most likely pathophysiology responsible for the patient’s findings?
Correct
Pancytopenia (a reduction in all blood cell lines: erythrocytes, leukocytes, and platelets) with a low reticulocyte count is suggestive of aplastic anemia. The use of medications known to cause bone marrow suppression further suggests drug-induced aplastic anemia. Bone marrow generates pluripotent stem cells that differentiate into red blood cells (erythrocytes), white blood cells (leukocytes), and platelets (thrombocytes). Immature red blood cells are called reticulocytes. When red blood cell production is increased, the erythrocyte count becomes elevated. If this level is reduced, concern is raised about the integrity of bone marrow stem cell production. When red blood cell counts are low (anemia), the hormone erythropoietin is released to accelerate red cell production. Aplastic anemia is a rare type of anemia that is caused by drugs or chemicals in 50% of cases. Drugs that suppress bone marrow production include chloramphenicol, anticonvulsants (e.g., carbamazepine), insecticides, sulfonamides, and gold. Medical conditions associated with aplastic anemia include viral hepatitis, radiation, pregnancy, and autoimmune diseases. Aplasia may affect a single cell line (e.g., pure red cell aplasia) or all cell lines. Symptoms of aplastic anemia may include fatigue, malaise, infection (from neutropenia), or mucosal bleeding (from thrombocytopenia). Bone marrow analysis is needed for definitive diagnosis.
Disseminated bacteremia (A) with certain organisms (e.g., Streptococcus pneumoniae, Neisseria gonorrhoeae) may present with purpura or petechiae. However, these patients are septic with fever, hypotension, lactic acidosis, and usually normal or elevated white blood cell counts and normal red blood cell counts. Disseminated intravascular coagulation (B) is a consumptive coagulopathy with multiple etiologies (e.g., sepsis, trauma, pregnancy, cancer) that can also present with excessive bleeding and petechiae. Thrombocytopenia and schistocytes are characteristic, as well as abnormal coagulation tests. Red blood cell and white blood cell lines are typically unaffected. A type I hypersensitivity reaction (D) is an immunoglobulin E (IgE)-mediated process that results in a massive, abrupt immune response affecting multiple organ systems (e.g., skin, respiratory, circulatory). Anaphylaxis is an example.
Incorrect
Pancytopenia (a reduction in all blood cell lines: erythrocytes, leukocytes, and platelets) with a low reticulocyte count is suggestive of aplastic anemia. The use of medications known to cause bone marrow suppression further suggests drug-induced aplastic anemia. Bone marrow generates pluripotent stem cells that differentiate into red blood cells (erythrocytes), white blood cells (leukocytes), and platelets (thrombocytes). Immature red blood cells are called reticulocytes. When red blood cell production is increased, the erythrocyte count becomes elevated. If this level is reduced, concern is raised about the integrity of bone marrow stem cell production. When red blood cell counts are low (anemia), the hormone erythropoietin is released to accelerate red cell production. Aplastic anemia is a rare type of anemia that is caused by drugs or chemicals in 50% of cases. Drugs that suppress bone marrow production include chloramphenicol, anticonvulsants (e.g., carbamazepine), insecticides, sulfonamides, and gold. Medical conditions associated with aplastic anemia include viral hepatitis, radiation, pregnancy, and autoimmune diseases. Aplasia may affect a single cell line (e.g., pure red cell aplasia) or all cell lines. Symptoms of aplastic anemia may include fatigue, malaise, infection (from neutropenia), or mucosal bleeding (from thrombocytopenia). Bone marrow analysis is needed for definitive diagnosis.
Disseminated bacteremia (A) with certain organisms (e.g., Streptococcus pneumoniae, Neisseria gonorrhoeae) may present with purpura or petechiae. However, these patients are septic with fever, hypotension, lactic acidosis, and usually normal or elevated white blood cell counts and normal red blood cell counts. Disseminated intravascular coagulation (B) is a consumptive coagulopathy with multiple etiologies (e.g., sepsis, trauma, pregnancy, cancer) that can also present with excessive bleeding and petechiae. Thrombocytopenia and schistocytes are characteristic, as well as abnormal coagulation tests. Red blood cell and white blood cell lines are typically unaffected. A type I hypersensitivity reaction (D) is an immunoglobulin E (IgE)-mediated process that results in a massive, abrupt immune response affecting multiple organ systems (e.g., skin, respiratory, circulatory). Anaphylaxis is an example.
This marks the beginning of the Hematology and Oncology Block, the first of the two weeks will be Hematology. We will start conference with a quiz review, followed by Dr. Vincent’s F/U rounds, followed by FLIP by the Dr. Bajkowski and Dr. Aquino. We will then wrap up with a resident only meeting.
You have already completed the quiz before. Hence you can not start it again.
Quiz is loading...
You must sign in or sign up to start the quiz.
You have to finish following quiz, to start this quiz:
Results
0 of 10 questions answered correctly
Your time:
Time has elapsed
You have reached 0 of 0 points, (0)
Average score
Your score
Categories
Not categorized0%
Infectious0%
1
2
3
4
5
6
7
8
9
10
Answered
Review
Question 1 of 10
1. Question
22 year old male presents with altered mental status. He has a nonblanching rash shown below. Vital signs are HR 114, BP 100/89, RR 16, Temp 40.8 C. He is wearing sunglasses in the ER and complains about a headache. What is the next step in his care?
Correct
Up to 30% of meningococcemia patients will present with this picture and no immediate evidence of meningitis or sepsis.
Antibiotics should not be held while waiting for a lumbar puncture. There have been multiple studies looking at antibiotics effects of lumbar puncture results. Here is a link to 3 articles looking at this.
Dexamethasone should also be started as it decreases neurological complications from step pneumoniae meningitis.
Incorrect
Up to 30% of meningococcemia patients will present with this picture and no immediate evidence of meningitis or sepsis.
Antibiotics should not be held while waiting for a lumbar puncture. There have been multiple studies looking at antibiotics effects of lumbar puncture results. Here is a link to 3 articles looking at this.
Dexamethasone should also be started as it decreases neurological complications from step pneumoniae meningitis.
Question 2 of 10
2. Question
A 31-year-old man presents with double vision and difficulty swallowing. This all started a few days ago after he ate a bottle of home canned peaches. He has no medical history and takes no medication. Your examination is remarkable for bilateral ptosis, mildly dilated pupils, dry mucous membranes, difficulty swallowing, and weakness of the trapezius and deltoid muscles. Bowel sounds are absent. Vital signs are temperature 36. 3° C, heart rate 93 beats/minute, blood pressure 84/63 mm Hg, and respiratory rate 20 breaths/minute. Which of the following is indicated?
Correct
Botulism presents as a descending paralysis/anticholinergic syndrome. Autonomic dysfunction with orthostatic hypotension is common. Ileus and urinary retention may occur. Antitoxin, intensive care unit admission, and early intubation are often indicated.
Incorrect
Botulism presents as a descending paralysis/anticholinergic syndrome. Autonomic dysfunction with orthostatic hypotension is common. Ileus and urinary retention may occur. Antitoxin, intensive care unit admission, and early intubation are often indicated.
Question 3 of 10
3. Question
A 62-year-old male farmer with no medical problems and unknown vaccination status presents with leg pain and muscle spasms. He reports moderate to severe pain and muscle spasms in the calf that have occurred and worsened during 3 days. Two weeks prior, he suffered a puncture wound to his ankle just above his boot top with a piece of metal. Examination is remarkable for a heart rate of 115 beats/minute, blood pressure of 170/110 mm Hg, and a healing clean wound above the medial malleolus with no evidence of active infection. The calf is in active spasm with some increased tone in the peroneal musculature also. Which is the next step in the patients treatment?
Correct
Localized tetanus reflects a local neuromuscular process with pain and spasm. It is likely due to a partial immunity. Immune globulin is indicated. Although mortality is lower, it can progress to generalized tetanus, and admission is warranted.
HTIG prophylaxis (250 units IM) is recommended for unimmunized and underimmunized patients with wounds at high risk for tetanus (>6 hours old, >1 cm deep, contaminated, stellate, denervated, ischemic, infected).27 When tetanus toxoid and HTIG are given concurrently, separate injection sites should be used. The only contraindication to tetanus and diphtheria toxoids is a history of a neurologic or severe hypersensitivity reaction to a previous dose. Adverse reactions to tetanus toxoid and tetanus-diphtheria toxoids occur commonly and may be the result of the preservative thimerosal. The most common side effects are minor: local swelling, pain, erythema, pruritus, fever, nausea, vomiting, malaise, and nonspecific rash.
Incorrect
Localized tetanus reflects a local neuromuscular process with pain and spasm. It is likely due to a partial immunity. Immune globulin is indicated. Although mortality is lower, it can progress to generalized tetanus, and admission is warranted.
HTIG prophylaxis (250 units IM) is recommended for unimmunized and underimmunized patients with wounds at high risk for tetanus (>6 hours old, >1 cm deep, contaminated, stellate, denervated, ischemic, infected).27 When tetanus toxoid and HTIG are given concurrently, separate injection sites should be used. The only contraindication to tetanus and diphtheria toxoids is a history of a neurologic or severe hypersensitivity reaction to a previous dose. Adverse reactions to tetanus toxoid and tetanus-diphtheria toxoids occur commonly and may be the result of the preservative thimerosal. The most common side effects are minor: local swelling, pain, erythema, pruritus, fever, nausea, vomiting, malaise, and nonspecific rash.
Question 4 of 10
4. Question
A 26-year-old woman presents two days after an operation for recurrent sinusitis. Her husband states that she has been confused since she got “the flu” yesterday. Her vitals are temperature 39.5°C, HR 115, BP 95/70, and oxygen saturation is 99% on room air. On exam, she is disoriented and has a diffusely hyperemic, blanching rash. She has a surgical dressing covering her nose. What is the next step in management?
Correct
The surgical dressing must be removed to ensure there is no nasal packing or other foreign body present that could serve as a precipitant of toxic shock syndrome (fever, hypotension, diffuse erythroderma, multisystem organ dysfunction). Toxic shock syndrome is commonly associated with postsurgical dressings as well as vaginal foreign bodies (classically, extended-use tampons). Toxic shock syndrome toxin-1 (TSST-1) producing strains of S. aureuscause the infection, with the toxin serving as a superantigen that leads to overstimulation of T-lymphocytes and subsequent massive, unregulated cytokine release. Patients often report a prodrome of flu-like symptoms including headache, myalgias, vomiting, and diarrhea.
Antibiotics (A) prevent recurrence but do not affect the outcome of the acute illness. However, starting an antistaphylococcal antibiotic early is standard practice. There is evidence that the addition of clindamycin or linezolid offers an additional benefit of decreasing production of TSST-1. A wound culture (C) has limited utility in this scenario. A blood culture may be useful but should be obtained after the nidus of infection is removed. All diagnostic testing, including imaging (B), should be performed after searching for and removing the likely source of toxin.
Incorrect
The surgical dressing must be removed to ensure there is no nasal packing or other foreign body present that could serve as a precipitant of toxic shock syndrome (fever, hypotension, diffuse erythroderma, multisystem organ dysfunction). Toxic shock syndrome is commonly associated with postsurgical dressings as well as vaginal foreign bodies (classically, extended-use tampons). Toxic shock syndrome toxin-1 (TSST-1) producing strains of S. aureuscause the infection, with the toxin serving as a superantigen that leads to overstimulation of T-lymphocytes and subsequent massive, unregulated cytokine release. Patients often report a prodrome of flu-like symptoms including headache, myalgias, vomiting, and diarrhea.
Antibiotics (A) prevent recurrence but do not affect the outcome of the acute illness. However, starting an antistaphylococcal antibiotic early is standard practice. There is evidence that the addition of clindamycin or linezolid offers an additional benefit of decreasing production of TSST-1. A wound culture (C) has limited utility in this scenario. A blood culture may be useful but should be obtained after the nidus of infection is removed. All diagnostic testing, including imaging (B), should be performed after searching for and removing the likely source of toxin.
Question 5 of 10
5. Question
An 85-year-old man is brought to the emergency department (ED) by his daughter, who says that he has had a fever and has been eating less than normal for 2 days. The patient has a history of hypertension and severe dementia. He can provide no useful information. His present temperature was recorded by the triage nurse to be 38. 5° C. The remainder of his vital signs are within normal limits. The only abnormal findings on physical examination are slightly dry mucous membranes, confusion, and poor social interaction. The daughter confirms his mental status to be at baseline. Which of the following are the two most important ancillary tests to obtain in this patient?
Correct
The two most important ancillary tests in the evaluation of fever in the adult patient, and especially in elder patients who frequently have atypical presentations, are urinalysis and chest radiography. Chest radiographs are often helpful in the diagnosis of pulmonary infection but may be difficult to interpret in the patient with chronic obstructive pulmonary disease (COPD), congestive heart failure (CHF), or other chronic lung disease. The urinalysis, although not foolproof, is highly accurate for urinary tract infection, especially in men. Although the white blood cell count is almost universally used in the evaluation of febrile patients, it lacks sensitivity and specificity to be of discriminatory value. Cultures are ordered in selected patients; however, the delay in obtaining results precludes any influence in emergency evaluation and treatment. Other tests that have relevance in select patients with fever include Gram’s stain, cerebrospinal fluid (CSF) analysis, thyroid function studies, ultrasonography, and computed tomography (CT) of the abdomen or head.
Incorrect
The two most important ancillary tests in the evaluation of fever in the adult patient, and especially in elder patients who frequently have atypical presentations, are urinalysis and chest radiography. Chest radiographs are often helpful in the diagnosis of pulmonary infection but may be difficult to interpret in the patient with chronic obstructive pulmonary disease (COPD), congestive heart failure (CHF), or other chronic lung disease. The urinalysis, although not foolproof, is highly accurate for urinary tract infection, especially in men. Although the white blood cell count is almost universally used in the evaluation of febrile patients, it lacks sensitivity and specificity to be of discriminatory value. Cultures are ordered in selected patients; however, the delay in obtaining results precludes any influence in emergency evaluation and treatment. Other tests that have relevance in select patients with fever include Gram’s stain, cerebrospinal fluid (CSF) analysis, thyroid function studies, ultrasonography, and computed tomography (CT) of the abdomen or head.
Question 6 of 10
6. Question
A 65-year-old man with a history of poorly controlled diabetes presents with a painful area of erythema on his anterior thigh which has been rapidly expanding over the last few hours. He is diagnosed with necrotizing fasciitis and taken to the operating room. Which of the following is most likely to be seen on tissue culture?
Correct
Most cases of necrotizing fasciitis are caused by polymicrobial infection with gram-negative, gram-positive, andanaerobic bacteria. Commonly identified organisms include S. aureus, beta-hemolytic streptococci, enterococci, enterobacteria, and anerobes such as Bacteroides and Clostridium species. Necrotizing fasciitis is characterized by deep tissue infection involving subcutaneous fat, fascia, and muscle layers, as opposed to cellulitis, where infection does not extend past the dermal layer. Infection may begin by direct bacterial invasion of subcutaneous tissue via breaks in the skin from an abscess or wound, or via spread from a perforated viscous such as colon, rectum, or anus. The hallmark of necrotizing fasciitis is pain out of proportion to physical exam. Early recognition is critical as necrotizing fasciitis is rapidly progressive and has a high mortality rate. General surgery should be consulted as soon as necrotizing fasciitis is suspected. It is treated with surgical debridement, intravenous broad-spectrum antibiotics, and hemodynamic support.
Necrotizing fasciitis of the perineal, perianal, or scrotal is called Fournier’s gangrene.
Incorrect
Most cases of necrotizing fasciitis are caused by polymicrobial infection with gram-negative, gram-positive, andanaerobic bacteria. Commonly identified organisms include S. aureus, beta-hemolytic streptococci, enterococci, enterobacteria, and anerobes such as Bacteroides and Clostridium species. Necrotizing fasciitis is characterized by deep tissue infection involving subcutaneous fat, fascia, and muscle layers, as opposed to cellulitis, where infection does not extend past the dermal layer. Infection may begin by direct bacterial invasion of subcutaneous tissue via breaks in the skin from an abscess or wound, or via spread from a perforated viscous such as colon, rectum, or anus. The hallmark of necrotizing fasciitis is pain out of proportion to physical exam. Early recognition is critical as necrotizing fasciitis is rapidly progressive and has a high mortality rate. General surgery should be consulted as soon as necrotizing fasciitis is suspected. It is treated with surgical debridement, intravenous broad-spectrum antibiotics, and hemodynamic support.
Necrotizing fasciitis of the perineal, perianal, or scrotal is called Fournier’s gangrene.
Question 7 of 10
7. Question
Which of the following statements regarding septic shock is TRUE?
Correct
Sepsis affects both myocardial function and peripheral vascular tone. The systemic vascular resistance is usually markedly depressed. Cardiac output is generally increased because of a compensatory tachycardia that can at least partially overcome the ventricular dilation and depressed ejection fraction. The myocardial effects are typically reversible.
Incorrect
Sepsis affects both myocardial function and peripheral vascular tone. The systemic vascular resistance is usually markedly depressed. Cardiac output is generally increased because of a compensatory tachycardia that can at least partially overcome the ventricular dilation and depressed ejection fraction. The myocardial effects are typically reversible.
Question 8 of 10
8. Question
A 2-day-old boy presents to the ED with fever for the past four hours. His birth history includes a normal spontaneous vaginal delivery at term. Parents report noticing that the child “felt warm,” and that he was having copious nasal secretions while feeding. On physical examination, the child appears lethargic, has mottled extremities, and is hot to the touch. Breath sounds are clear bilaterally and there are no rashes. His vital signs are T 102.9°F, BP 74/48 mm Hg, HR 170 beats per minute, and RR 40 breaths per minute. Which of the following groupings of organisms should covered by your antibiotic choices when treating this febrile neonate?
Correct
The febrile neonate is a child 28 days and younger who presents with a fever. These children are at very high risk of serious bacterial infections including urinary tract infection, pneumonia, meningitis, and bacteremia. Risk factors for serious bacterial infection in a neonate include prematurity, low birth weight, premature or prolonged rupture of membranes, meconium aspiration, or maternal group B streptococcus infection. The evaluation of a neonate with a fever includes CBC, urinalysis, blood culture, urine culture, and a lumbar puncture in order to obtain CSF for cell count, gram stain, and culture. If the child has respiratory symptoms, a chest X-ray should be performed. If the child has diarrhea, stool testing should also be performed. The most common pathogens involved in serious bacterial infections, including meningitis and bacteremia, in neonates are Listeria monocytogenes, Group B streptococcus, and Escherichia coli. These children can become critically ill very rapidly; therefore, initial management should include a fluid bolus of 20 mL/kg and broad-spectrum antibiotics to cover for the most common pathogens in this age group. The most appropriate antibiotics to use in neonates with a fever are ampicillin and cefotaxime. Ampicillin will cover Listeria monocytogenes while cefotaxime will cover Group B streptococcus and Escherichia coli. If there is a history of maternal infection with herpes simplex virus, acyclovir should be added to the empiric broad-spectrum treatment. These patients universally need to be admitted to the hospital for IV antibiotics and observation until all cultures have returned.
Incorrect
The febrile neonate is a child 28 days and younger who presents with a fever. These children are at very high risk of serious bacterial infections including urinary tract infection, pneumonia, meningitis, and bacteremia. Risk factors for serious bacterial infection in a neonate include prematurity, low birth weight, premature or prolonged rupture of membranes, meconium aspiration, or maternal group B streptococcus infection. The evaluation of a neonate with a fever includes CBC, urinalysis, blood culture, urine culture, and a lumbar puncture in order to obtain CSF for cell count, gram stain, and culture. If the child has respiratory symptoms, a chest X-ray should be performed. If the child has diarrhea, stool testing should also be performed. The most common pathogens involved in serious bacterial infections, including meningitis and bacteremia, in neonates are Listeria monocytogenes, Group B streptococcus, and Escherichia coli. These children can become critically ill very rapidly; therefore, initial management should include a fluid bolus of 20 mL/kg and broad-spectrum antibiotics to cover for the most common pathogens in this age group. The most appropriate antibiotics to use in neonates with a fever are ampicillin and cefotaxime. Ampicillin will cover Listeria monocytogenes while cefotaxime will cover Group B streptococcus and Escherichia coli. If there is a history of maternal infection with herpes simplex virus, acyclovir should be added to the empiric broad-spectrum treatment. These patients universally need to be admitted to the hospital for IV antibiotics and observation until all cultures have returned.
Question 9 of 10
9. Question
A 65-year-old man with a history of poorly controlled diabetes presents with a painful area of erythema on his anterior thigh which has been rapidly expanding over the last few hours. He is diagnosed with necrotizing fasciitis and taken to the operating room. Which of the following is most likely to be seen on tissue culture?
Correct
Most cases of necrotizing fasciitis are caused by polymicrobial infection with gram-negative, gram-positive, andanaerobic bacteria. Commonly identified organisms include S. aureus, beta-hemolytic streptococci, enterococci, enterobacteria, and anerobes such as Bacteroides and Clostridium species. Necrotizing fasciitis is characterized by deep tissue infection involving subcutaneous fat, fascia, and muscle layers, as opposed to cellulitis, where infection does not extend past the dermal layer. Infection may begin by direct bacterial invasion of subcutaneous tissue via breaks in the skin from an abscess or wound, or via spread from a perforated viscous such as colon, rectum, or anus. The hallmark of necrotizing fasciitis is pain out of proportion to physical exam. Early recognition is critical as necrotizing fasciitis is rapidly progressive and has a high mortality rate. General surgery should be consulted as soon as necrotizing fasciitis is suspected. It is treated with surgical debridement, intravenous broad-spectrum antibiotics, and hemodynamic support.
Necrotizing fasciitis of the perineal, perianal, or scrotal is called Fournier’s gangrene.
Incorrect
Most cases of necrotizing fasciitis are caused by polymicrobial infection with gram-negative, gram-positive, andanaerobic bacteria. Commonly identified organisms include S. aureus, beta-hemolytic streptococci, enterococci, enterobacteria, and anerobes such as Bacteroides and Clostridium species. Necrotizing fasciitis is characterized by deep tissue infection involving subcutaneous fat, fascia, and muscle layers, as opposed to cellulitis, where infection does not extend past the dermal layer. Infection may begin by direct bacterial invasion of subcutaneous tissue via breaks in the skin from an abscess or wound, or via spread from a perforated viscous such as colon, rectum, or anus. The hallmark of necrotizing fasciitis is pain out of proportion to physical exam. Early recognition is critical as necrotizing fasciitis is rapidly progressive and has a high mortality rate. General surgery should be consulted as soon as necrotizing fasciitis is suspected. It is treated with surgical debridement, intravenous broad-spectrum antibiotics, and hemodynamic support.
Necrotizing fasciitis of the perineal, perianal, or scrotal is called Fournier’s gangrene.
Question 10 of 10
10. Question
Which of the following is the highest severity sepsis syndrome met by the following patient: 87-year-old male with diabetes, pneumonia, heart rate 110 beats per minute, temperature 101.6°F, creatinine 3.4 mg/dL (baseline is 0.8 mg/dL), and blood pressure 120/80 mm Hg?
Correct
That’s correct!
Sepsis is defined as meeting systemic inflammatory response criteria (SIRS) plus a suspected infection. Severe sepsis is sepsis plus organ dysfunction. Septic shock is sepsis plus hypotension not responsive to a fluid challenge. This patient has SIRS (elevated heart rate and elevated temperature), renal dysfunction (new creatinine elevation), but a normal blood pressure; thus, he meets the definition for severe sepsis.
Incorrect
That’s correct!
Sepsis is defined as meeting systemic inflammatory response criteria (SIRS) plus a suspected infection. Severe sepsis is sepsis plus organ dysfunction. Septic shock is sepsis plus hypotension not responsive to a fluid challenge. This patient has SIRS (elevated heart rate and elevated temperature), renal dysfunction (new creatinine elevation), but a normal blood pressure; thus, he meets the definition for severe sepsis.
We will start the day with Dr. McElroy’s follow up rounds, a special guest sepsis lecture, and then some hearty Flip by Drs. Matt McRae, and Brad Smith. This will be followed by M&M with our very own Dr. Loftus. This will be followed by a special M&M with the good Dr. Loftus.
You have already completed the quiz before. Hence you can not start it again.
Quiz is loading...
You must sign in or sign up to start the quiz.
You have to finish following quiz, to start this quiz:
Results
0 of 10 questions answered correctly
Your time:
Time has elapsed
You have reached 0 of 0 points, (0)
Average score
Your score
Categories
Not categorized0%
1
2
3
4
5
6
7
8
9
10
Answered
Review
Question 1 of 10
1. Question
A 24-year-old man presents to the ED after a syncopal episode. He is an avid hiker. He had a rash a few weeks prior that has since cleared. Vital signs are BP 100/74 mm Hg, HR 50 beats/minute, RR 18 breaths/minute, and T 99.1°F. What is the most likely finding on this patient’s electrocardiogram?
Correct
This patient is exhibiting signs and symptoms of Lyme disease. Lyme disease is the most common vector-borne disease in the United States. It is endemic to New England, the mid-Atlantic states, and the upper Midwest. It is caused by the spirochete Borrelia burgdorferi and transmitted by the Ixodes dammini tick, more commonly known as the deer tick. The tick must be attached for more than 48 hours for transmission to occur. There are three stages of clinical Lyme disease. Early Lyme disease is characterized by erythema migrans, an erythematous blanching patch that may have central clearing and classically has a “bull’s eye” appearance. Hematogenous spread leads to diffuse erythema migrans, which spares the palms and soles. Acute disseminated Lyme disease occurs approximately 4 weeks after initial infection and can include meningoencephalitis, Bell’s palsy (which may be bilateral), or carditis which often manifests with variable AV block. This AV blockade may be high-grade and require temporary transcutaneous or transvenous pacing; however, it nearly universally resolves gradually with IV antibiotics and has a favorable prognosis with no need for permanent pacemaker placement. Late Lyme disease develops greater than 1 year after initial infection and includes chronic arthritis with or without chronic subtle encephalopathy. Only 50% of patients remember a tick bite; thus, diagnosis may be difficult. Erythema migrans is diagnostic; however, not all patients present with this finding. Initial screening involves ELISA testing with Western Blot and PCR to confirm the diagnosis. If the diagnosis is suspected, empiric treatment should be administered. Treatment for early Lyme disease and mild acute disseminated Lyme disease is doxycycline for 3-4 weeks. In pregnant women or children under the age of 8 years, amoxicillin should be substituted. Patients with neurologic or cardiac manifestations should be admitted and treated with IV ceftriaxone.
Incorrect
This patient is exhibiting signs and symptoms of Lyme disease. Lyme disease is the most common vector-borne disease in the United States. It is endemic to New England, the mid-Atlantic states, and the upper Midwest. It is caused by the spirochete Borrelia burgdorferi and transmitted by the Ixodes dammini tick, more commonly known as the deer tick. The tick must be attached for more than 48 hours for transmission to occur. There are three stages of clinical Lyme disease. Early Lyme disease is characterized by erythema migrans, an erythematous blanching patch that may have central clearing and classically has a “bull’s eye” appearance. Hematogenous spread leads to diffuse erythema migrans, which spares the palms and soles. Acute disseminated Lyme disease occurs approximately 4 weeks after initial infection and can include meningoencephalitis, Bell’s palsy (which may be bilateral), or carditis which often manifests with variable AV block. This AV blockade may be high-grade and require temporary transcutaneous or transvenous pacing; however, it nearly universally resolves gradually with IV antibiotics and has a favorable prognosis with no need for permanent pacemaker placement. Late Lyme disease develops greater than 1 year after initial infection and includes chronic arthritis with or without chronic subtle encephalopathy. Only 50% of patients remember a tick bite; thus, diagnosis may be difficult. Erythema migrans is diagnostic; however, not all patients present with this finding. Initial screening involves ELISA testing with Western Blot and PCR to confirm the diagnosis. If the diagnosis is suspected, empiric treatment should be administered. Treatment for early Lyme disease and mild acute disseminated Lyme disease is doxycycline for 3-4 weeks. In pregnant women or children under the age of 8 years, amoxicillin should be substituted. Patients with neurologic or cardiac manifestations should be admitted and treated with IV ceftriaxone.
Question 2 of 10
2. Question
A 21-year-old man presents with a rash and fever. He was recently camping in North Carolina. He noted a rash that began on his ankles and wrists that is now present on his chest and back. Which of the following is the most likely diagnosis?
Correct
Rocky Mountain spotted fever (RMSF) is a tick-borne illness caused by Rickettsia rickettsii and transmitted to humans through the dog tick or Rocky Mountain wood tick. The disease is endemic in all 48 states, but most commonly occurs in thesoutheastern United States. After infection, the organism invades and replicates inside of vascular endothelial cells. The most common victims of RMSF are children. After a mean incubation period of 7 days, patients develop classic symptoms including fever, headache, myalgias, nausea and vomiting. Fever is usually the first symptom and may be the only symptom for up to one week. The rash is classically small red macules that are sometimes pruritic. The rash begins only on the ankles and feet and then spreads to the palms and soles. Thereafter, it moves centrally to the rest of the arms, legs and torso. After several days, the rash becomes papular and deeper red. In the early stage the rash blanches with direct palpation and enhances with warm compresses. In later stages, the rash becomes petechial and application of proximal pressure causes additional lesions to appear distally (Rumpel-Leede phenomenon). The cardiovascular system is affected with myocarditis and an impaired ejection fraction in some patients. Neurologically, the most common symptom is headache but may include seizures and an eosinophilic meningitis. Diagnosis is made through the identification of serum antibodies present 5 to 7 days after the infection begins. Treatment is with doxycycline
Babesiosis (A) is a tick-borne illnesses transmitted by the Ixodes tick in the northeastern part of the United States. The clinical symptoms are similar to malaria as the organism infects red blood cells causing fevers, myalgias headache and significant sweats. Approximately 20% of patients with babesiosis are co-infected with Lyme disease. Leptospirosis (B)is most commonly transmitted to humans through the urine of rodents. In most cases, patients have a flu-like illness with fever, myalgia and severe headache. Ten percent of patients develop severe leptospirosis with liver and renal failure.Lyme disease (C) is the most common vector-borne disease in the United States. The Ixodes ticks transmit the organism Borrelia burgdorferi. The initial manifestation of Lyme disease is the classic rash erythema migrans described as an erythematous macule with central clearing. With time patients develop fever, arthralgias, adenopathy and ultimately arthritis. Neurologically patients may develop unilateral or bilateral Bell’s palsy, aseptic meningitis and a neuritis causing similar pain symptoms to post-herpetic neuralgia.
Incorrect
Rocky Mountain spotted fever (RMSF) is a tick-borne illness caused by Rickettsia rickettsii and transmitted to humans through the dog tick or Rocky Mountain wood tick. The disease is endemic in all 48 states, but most commonly occurs in thesoutheastern United States. After infection, the organism invades and replicates inside of vascular endothelial cells. The most common victims of RMSF are children. After a mean incubation period of 7 days, patients develop classic symptoms including fever, headache, myalgias, nausea and vomiting. Fever is usually the first symptom and may be the only symptom for up to one week. The rash is classically small red macules that are sometimes pruritic. The rash begins only on the ankles and feet and then spreads to the palms and soles. Thereafter, it moves centrally to the rest of the arms, legs and torso. After several days, the rash becomes papular and deeper red. In the early stage the rash blanches with direct palpation and enhances with warm compresses. In later stages, the rash becomes petechial and application of proximal pressure causes additional lesions to appear distally (Rumpel-Leede phenomenon). The cardiovascular system is affected with myocarditis and an impaired ejection fraction in some patients. Neurologically, the most common symptom is headache but may include seizures and an eosinophilic meningitis. Diagnosis is made through the identification of serum antibodies present 5 to 7 days after the infection begins. Treatment is with doxycycline
Babesiosis (A) is a tick-borne illnesses transmitted by the Ixodes tick in the northeastern part of the United States. The clinical symptoms are similar to malaria as the organism infects red blood cells causing fevers, myalgias headache and significant sweats. Approximately 20% of patients with babesiosis are co-infected with Lyme disease. Leptospirosis (B)is most commonly transmitted to humans through the urine of rodents. In most cases, patients have a flu-like illness with fever, myalgia and severe headache. Ten percent of patients develop severe leptospirosis with liver and renal failure.Lyme disease (C) is the most common vector-borne disease in the United States. The Ixodes ticks transmit the organism Borrelia burgdorferi. The initial manifestation of Lyme disease is the classic rash erythema migrans described as an erythematous macule with central clearing. With time patients develop fever, arthralgias, adenopathy and ultimately arthritis. Neurologically patients may develop unilateral or bilateral Bell’s palsy, aseptic meningitis and a neuritis causing similar pain symptoms to post-herpetic neuralgia.
Question 3 of 10
3. Question
Which of the following statements regarding AIDS and parasitic infections is TRUE?
Correct
Isospora and coccidial organisms may cause an almost cholera-like diarrheal illness. Eradication is very difficult.
Malaria and invasive amebiasis are not considered opportunistic infections.
AIDS patients have much more severe allergic manifestations to the antiparasitics.
Schistosomiasis enhances HIV pathogenesis.
Incorrect
Isospora and coccidial organisms may cause an almost cholera-like diarrheal illness. Eradication is very difficult.
Malaria and invasive amebiasis are not considered opportunistic infections.
AIDS patients have much more severe allergic manifestations to the antiparasitics.
Schistosomiasis enhances HIV pathogenesis.
Question 4 of 10
4. Question
A macrocytic anemia would suggest infection from which parasite?
Correct
The fish tapeworm is associated with pernicious anemia. Hookworm and whipworm are associated with gastrointestinal iron loss and microcytic anemia. Malaria causes hemolytic anemia.
Incorrect
The fish tapeworm is associated with pernicious anemia. Hookworm and whipworm are associated with gastrointestinal iron loss and microcytic anemia. Malaria causes hemolytic anemia.
Question 5 of 10
5. Question
Parasite-induced loss of vision would be suggested by which of the following?
Correct
Onchocerciasis is a major cause of blindness worldwide. Ninety-five percent of cases occur in Africa. The biting flies are found near rivers, and humans are the only host for the parasite. It occupies the skin, resulting in pruritus, edema, and later atrophy with redundant skin folds. The following are other causes of parasite-induced visual loss: Toxoplasma can cause retinal hemorrhages, Toxocara can cause inflammatory retinal granulomas, and Acanthamoeba may cause a keratitis in contact lens wearers.
Incorrect
Onchocerciasis is a major cause of blindness worldwide. Ninety-five percent of cases occur in Africa. The biting flies are found near rivers, and humans are the only host for the parasite. It occupies the skin, resulting in pruritus, edema, and later atrophy with redundant skin folds. The following are other causes of parasite-induced visual loss: Toxoplasma can cause retinal hemorrhages, Toxocara can cause inflammatory retinal granulomas, and Acanthamoeba may cause a keratitis in contact lens wearers.
Question 6 of 10
6. Question
A 45-year-old man with HIV-AIDS and a CD4 count of 150 cells/mm3 presents with several days of fever and dry cough. His oxygen saturation is 85% on room air. Lung auscultation is notable for decreased breath sounds bilaterally and crackles at the bases. A chest X-ray reveals bilateral interstitial infiltrates. Which of the following medication regimens is most appropriate?
Correct
This patient has clinical and radiographic evidence for pneumonia. Community-acquired pneumonia caused by Streptococcus pneumoniae is the most common cause of pneumonia in HIV-infected patients. Therefore, he should receive ceftriaxone and azithromycin. Additionally, his low CD4 count puts him at risk of opportunistic infections, such as Pneumocystis jiroveci pneumonia (PJP), formerly known as Pneumocystis carinii pneumonia (PCP). PJP is the most common opportunistic infection among AIDS patients. Signs and symptoms of PJP pneumonia include fatigue, fever, cough, and hypoxia. Chest radiographs typically show diffuse interstitial infiltrates, though negative radiographs are not unusual in PJP. Serum lactate dehydrogenase levels are often elevated. Initial therapy for PJP includes trimethoprim/sulfamethoxazole. In severe cases, corticosteroids are also given. The indications for corticosteroid use include an arterial oxygen partial pressure of < 70 mm Hg (equivalent to an oxygen saturation of <~93%) and an alveolar-arterial gradient > 35 mm Hg. Prophylaxis with oral trimethoprim-sulfamethoxazole is recommended for all HIV-positive patients with CD4 counts less than 200 cells/mm3 to prevent PJP pneumonia.
Amphotericin B (A) is used in the treatment of disseminated fungal infections, such as Aspergillus fumigatus and Cryptococcus neoformans. These infections are typically seen at a higher degree of immunosuppression. Azithromycin and ceftriaxone (B) treat most community-acquired pathogens, but do not cover opportunistic infections like PJP. Trimethoprim/sulfamethoxazole and prednisone (D) is incorrect because the patient should also receive treatment for community-acquired pneumonia.
Incorrect
This patient has clinical and radiographic evidence for pneumonia. Community-acquired pneumonia caused by Streptococcus pneumoniae is the most common cause of pneumonia in HIV-infected patients. Therefore, he should receive ceftriaxone and azithromycin. Additionally, his low CD4 count puts him at risk of opportunistic infections, such as Pneumocystis jiroveci pneumonia (PJP), formerly known as Pneumocystis carinii pneumonia (PCP). PJP is the most common opportunistic infection among AIDS patients. Signs and symptoms of PJP pneumonia include fatigue, fever, cough, and hypoxia. Chest radiographs typically show diffuse interstitial infiltrates, though negative radiographs are not unusual in PJP. Serum lactate dehydrogenase levels are often elevated. Initial therapy for PJP includes trimethoprim/sulfamethoxazole. In severe cases, corticosteroids are also given. The indications for corticosteroid use include an arterial oxygen partial pressure of < 70 mm Hg (equivalent to an oxygen saturation of <~93%) and an alveolar-arterial gradient > 35 mm Hg. Prophylaxis with oral trimethoprim-sulfamethoxazole is recommended for all HIV-positive patients with CD4 counts less than 200 cells/mm3 to prevent PJP pneumonia.
Amphotericin B (A) is used in the treatment of disseminated fungal infections, such as Aspergillus fumigatus and Cryptococcus neoformans. These infections are typically seen at a higher degree of immunosuppression. Azithromycin and ceftriaxone (B) treat most community-acquired pathogens, but do not cover opportunistic infections like PJP. Trimethoprim/sulfamethoxazole and prednisone (D) is incorrect because the patient should also receive treatment for community-acquired pneumonia.
Question 7 of 10
7. Question
Which of the following best describes the most common symptoms of acute human immunodeficiency virus infection?
Correct
In the United States, an estimated 1.2 million individuals are HIV positive, with approximately 50,000 people newly diagnosed each year. The clinical presentation of acute HIV infection, also known as acute retroviral syndrome, is variable both in type and severity of symptoms. The classic presentation is abrupt onset of a viral-like illness, which develops 10–14 days following exposure to the virus. Symptoms may include fever, malaise, sore throat, headache, arthralgias, anorexia, nausea, vomiting, and rash. Physical examination findings vary and may include generalized lymphadenopathy, nonexudative pharyngitis, mild hepatosplenomegaly, mucocutaneous ulcers, and oral thrush. A maculopapular rash on the thorax, face, and limbs may also be present. Due to the nonspecific and self-limited nature of symptoms, many affected individuals do not seek care and thus the true incidence of acute HIV is unknown. However, experts estimate that 60–90% of individuals who contract HIV develop an acute illness. Identification of acute HIV has tremendous public health implications, as acute infection represents the peak infectivity of the disease, characterized by high levels of viral shedding. Routine HIV tests used in most emergency departments measure anti-HIV antibodies, which are often falsely negative during acute infection. Therefore, when acute HIV is suspected, nucleic acid amplification tests, which directly measure viral RNA in the patient’s blood, should be utilized.
Incorrect
In the United States, an estimated 1.2 million individuals are HIV positive, with approximately 50,000 people newly diagnosed each year. The clinical presentation of acute HIV infection, also known as acute retroviral syndrome, is variable both in type and severity of symptoms. The classic presentation is abrupt onset of a viral-like illness, which develops 10–14 days following exposure to the virus. Symptoms may include fever, malaise, sore throat, headache, arthralgias, anorexia, nausea, vomiting, and rash. Physical examination findings vary and may include generalized lymphadenopathy, nonexudative pharyngitis, mild hepatosplenomegaly, mucocutaneous ulcers, and oral thrush. A maculopapular rash on the thorax, face, and limbs may also be present. Due to the nonspecific and self-limited nature of symptoms, many affected individuals do not seek care and thus the true incidence of acute HIV is unknown. However, experts estimate that 60–90% of individuals who contract HIV develop an acute illness. Identification of acute HIV has tremendous public health implications, as acute infection represents the peak infectivity of the disease, characterized by high levels of viral shedding. Routine HIV tests used in most emergency departments measure anti-HIV antibodies, which are often falsely negative during acute infection. Therefore, when acute HIV is suspected, nucleic acid amplification tests, which directly measure viral RNA in the patient’s blood, should be utilized.
Question 8 of 10
8. Question
A 39-year-old man presents to the ED with the findings seen in the above image. The lesions cannot be scraped off the surface of his tongue. Which of the following statements best describes this patient’s diagnosis?
Correct
The tongue lesions seen in the image are characteristic for oral hairy leukoplakia(OHL). This is a disease of the lingual squamous epithelium and is thought to be caused by the Epstein-Barr virus (EBV). It is highly specific for concomitant HIV infection and rarely seen in other immunocompromised states. OHL is often confused with oral candidiasis (thrush). However, unlike the lesions of thrush, OHL cannot be scraped off the tongue.
OHL most commonly develops on the lateral portions of the tongue, not the posterior oropharynx (A). The disease is caused by the Epstein-Barr virus, unlike oral thrush, which is caused by the yeast (C)Candida albicans. The plaques are painless (D), and most patients are asymptomatic.
Incorrect
The tongue lesions seen in the image are characteristic for oral hairy leukoplakia(OHL). This is a disease of the lingual squamous epithelium and is thought to be caused by the Epstein-Barr virus (EBV). It is highly specific for concomitant HIV infection and rarely seen in other immunocompromised states. OHL is often confused with oral candidiasis (thrush). However, unlike the lesions of thrush, OHL cannot be scraped off the tongue.
OHL most commonly develops on the lateral portions of the tongue, not the posterior oropharynx (A). The disease is caused by the Epstein-Barr virus, unlike oral thrush, which is caused by the yeast (C)Candida albicans. The plaques are painless (D), and most patients are asymptomatic.
Question 9 of 10
9. Question
12 year old presents to the emergency department with sudden fatigue and shortness of breath. He has a history of sickle cell disease. HR 125, BP 110/86, RR 18, temp. 38.6. Physical exam is significant for finding shown in picture below. IV access is obtained and labs are sent. What is the cause of the patients condition?
Correct
Aplastic crisis is defined as having a reticulocyte count of less than 1%. Patients with sickle cell disease who are infected with Parvovirus B19 are at risk for developing an aplastic crisis. Parvovirus B19 is the causative agent of Fifth disease and can lead to an aplastic crisis by causing a temporary arrest of red blood cell production. This is characterized by a sudden decrease in hemoglobin production by bone marrow resulting in severe anemia. Patients may present with pallor, lethargy, and shock. Treatment includes hemodynamic support and blood transfusion.
Labs that should be sent include type and cross, CBC, and reticulocyte count. This is usually self-limited disease and resolves once the viral syndrome resolves. Blood transfusions to support the patient through the anemia is the treatment along with other supportive measures.
Incorrect
Aplastic crisis is defined as having a reticulocyte count of less than 1%. Patients with sickle cell disease who are infected with Parvovirus B19 are at risk for developing an aplastic crisis. Parvovirus B19 is the causative agent of Fifth disease and can lead to an aplastic crisis by causing a temporary arrest of red blood cell production. This is characterized by a sudden decrease in hemoglobin production by bone marrow resulting in severe anemia. Patients may present with pallor, lethargy, and shock. Treatment includes hemodynamic support and blood transfusion.
Labs that should be sent include type and cross, CBC, and reticulocyte count. This is usually self-limited disease and resolves once the viral syndrome resolves. Blood transfusions to support the patient through the anemia is the treatment along with other supportive measures.
Question 10 of 10
10. Question
23 year old presents to the emergency department with a rash. They state that they were diagnosed with strep throat 5 days ago. They recently started on antibiotics which they do not know the name of. Yesterday they started to breakout in a rash which is shown below. What is the cause of the rash?
Correct
Infectious mononucleosis is caused by the Epstein-Barr virus. While young children with the disease typically have minimal, if any, symptoms related to the infection, adolescents and young adults usually have a more pronounced course. It is uncommonly seen in adults as most are immune due to a previous exposure. Patients may have a prodrome of low-grade fever, headache, and malaise. There is pharyngeal erythema and tonsillar exudates that may appear white, gray-green, or necrotic. Palatal petechiae can also be seen, but may be present in streptococcal pharyngitis as well. Severe fatigue is common and often the symptom that persists the longest. The lymphadenopathy is usually mildly tender, symmetric and involves the posterior cervical chain. Hepatosplenomegaly can be noted on examination. A generalized maculopapular rash almost always occurs following administration of amoxicillin or ampicillin although the mechanism of this is unclear. Diagnosis is made based on history and physical and can be confirmed with a positive heterophile antibody test (monospot test). Treatment is supportive. Symptoms generally resolve in 1-3 weeks although the fatigue may persist for months. Patients should refrain from contact sports for four weeks post-infection.
Incorrect
Infectious mononucleosis is caused by the Epstein-Barr virus. While young children with the disease typically have minimal, if any, symptoms related to the infection, adolescents and young adults usually have a more pronounced course. It is uncommonly seen in adults as most are immune due to a previous exposure. Patients may have a prodrome of low-grade fever, headache, and malaise. There is pharyngeal erythema and tonsillar exudates that may appear white, gray-green, or necrotic. Palatal petechiae can also be seen, but may be present in streptococcal pharyngitis as well. Severe fatigue is common and often the symptom that persists the longest. The lymphadenopathy is usually mildly tender, symmetric and involves the posterior cervical chain. Hepatosplenomegaly can be noted on examination. A generalized maculopapular rash almost always occurs following administration of amoxicillin or ampicillin although the mechanism of this is unclear. Diagnosis is made based on history and physical and can be confirmed with a positive heterophile antibody test (monospot test). Treatment is supportive. Symptoms generally resolve in 1-3 weeks although the fatigue may persist for months. Patients should refrain from contact sports for four weeks post-infection.
In-service exam is over! Congratulations! Thanks everyone for your hard work. I know everyone is stoked to take this quiz, but don’t worry, we are now getting back to some real medicine. Welcome back to FLIP!
The comeback starts with a 2 week block of Infectious Disease. This week, don’t miss our special guest lecture right at 8am – Pharmacy will be given an EM catered talk on infectious disease focusing on antibiotics!
Mark and Amanda then will have a brief hands-on nerve block lecture. We then will have FLIP by Drs Warpinski and Vandenberg. This will be followed up by M&M by Dr. Vaizer.
This is another week with a lot of various topics that is difficult to cover with FOAM, and we would recommend H&N or the two EBM articles. However, EMRAP does a really nice job going over HIV on their C3 podcast, and a general boards-focused overview with their crunch time (both linked below). If you can only get one chapter in, read 191.
The next 3 weeks of conference will be dedicated to the inservice exam, which is on February 26th. We will have board reviews for each conference, including other content. It is now crunch time — hit up Rosh, HippoEM, PEER, or any board-oriented source the best you can. If you have any trouble accessing resources, please reach out to the chiefs. No quizzes during these weeks, just do as many questions/review as you can before conference to be ready for the review. Good luck!
You have already completed the quiz before. Hence you can not start it again.
Quiz is loading...
You must sign in or sign up to start the quiz.
You have to finish following quiz, to start this quiz:
Results
0 of 10 questions answered correctly
Your time:
Time has elapsed
You have reached 0 of 0 points, (0)
Average score
Your score
Categories
Not categorized0%
1
2
3
4
5
6
7
8
9
10
Answered
Review
Question 1 of 10
1. Question
An 18 year old male is brought to a level 1 trauma center by ambulance with a stab wound to the right abdomen. Past medical history noncontributory. Vital signs are: BP 100/60 HR 85 RR 16 Temp 98.6F (37C). Exam reveals a diffuse tender abdomen with mild guarding and no rebound. A small amount of mesentery is noted to be extravasating from the wound. Which of the following is the next best step in management?
Correct
The patient has a penetrating abdominal injury with evisceration of abdominal contents. Indications for the OR in penetrating abdominal injury include unstable vital signs, peritonitis w/ a +FAST examination, evisceration, and transabdominal GSW.
Incorrect
The patient has a penetrating abdominal injury with evisceration of abdominal contents. Indications for the OR in penetrating abdominal injury include unstable vital signs, peritonitis w/ a +FAST examination, evisceration, and transabdominal GSW.
Question 2 of 10
2. Question
A 24-year-old man presents after being struck by a motor vehicle while crossing the street. His secondary survey is notable for an unstable pelvis which is stabilized with a pelvic binder. His initial and repeat FAST examinations in Tredelenberg are negative but he remains hypotensive and tachycardic. Intravenous fluids and uncrossmatched blood are administered. Which of the following is the most appropriate next step?
Correct
Patients with fractures of the bony pelvis are at high risk for hemorrhage due to the close proximity of the vascular structures to the pelvic bones. The internal iliac arteries and their subsequent branches provide most of the blood supply to the pelvic region. Most traumatic pelvic bleeding is venous, but in unstable patients an arterial source is suggested. In this patient with fractures and persistent hemodynamic instability without hemoperitoneum, the source of bleeding is likely the pelvis. After initial attempted stabilization with an external pelvic binder (either a sheet or commercial device), the patient needs angiography and embolization in interventional radiology. Angiography has very high success rates at controlling pelvic hemorrhage.
Incorrect
Patients with fractures of the bony pelvis are at high risk for hemorrhage due to the close proximity of the vascular structures to the pelvic bones. The internal iliac arteries and their subsequent branches provide most of the blood supply to the pelvic region. Most traumatic pelvic bleeding is venous, but in unstable patients an arterial source is suggested. In this patient with fractures and persistent hemodynamic instability without hemoperitoneum, the source of bleeding is likely the pelvis. After initial attempted stabilization with an external pelvic binder (either a sheet or commercial device), the patient needs angiography and embolization in interventional radiology. Angiography has very high success rates at controlling pelvic hemorrhage.
Question 3 of 10
3. Question
A 55-year old male presents to the ER after being struck by a car traveling approximately twenty miles per hour. Blood pressure is 140/90, heart rate is 105, respiratory rate is 16, and the patient is afebrile. Physical exam is significant for severe pain with manipulation of the pelvis as well as blood at the urethral meatus. X-ray is shown below. In addition to orthopedics consult, which of the following is the most appropriate next step in management?
Correct
The correct answer is retrograde urethrogram to evaluate for urethral injury. Posterior urethral injuries can occur with pelvic fractures. A bladder fracture is also a possibility and a retrograde cystogram should be performed after the urethrogram to assess for bladder injury given the open book pelvic fracture on x-ray. A foley catheter is contraindicated until urethral injury has been excluded. IVP is rarely performed currently as CT scan can evaluate for renal injury reliably. Retrograde cystogram should be done as well but after a retrograde urethrogram. Intravenous pyelography may be done if suspect kidney injury but blood at the urethral meatus in the context of a pelvic fracture is more suggestive of a urethral or bladder injury. Placement of a foley catheter is contraindicated
Incorrect
The correct answer is retrograde urethrogram to evaluate for urethral injury. Posterior urethral injuries can occur with pelvic fractures. A bladder fracture is also a possibility and a retrograde cystogram should be performed after the urethrogram to assess for bladder injury given the open book pelvic fracture on x-ray. A foley catheter is contraindicated until urethral injury has been excluded. IVP is rarely performed currently as CT scan can evaluate for renal injury reliably. Retrograde cystogram should be done as well but after a retrograde urethrogram. Intravenous pyelography may be done if suspect kidney injury but blood at the urethral meatus in the context of a pelvic fracture is more suggestive of a urethral or bladder injury. Placement of a foley catheter is contraindicated
Question 4 of 10
4. Question
A 42-year-old man was a restrained driver in a motor vehicle collision. He complains of left flank pain and is hemodynamically stable. Which of the following is an indication for a CT scan of the abdomen and pelvis with IV contrast?
Correct
The presence of gross hematuria in the setting of trauma signifies a urologic injury until proven otherwise. Gross hematuria suggests either a lower or upper urinary tract injury. Bladder injury (e.g. rupture) is suggested by lower abdominal pain and tenderness or a positive FAST examination on evaluation of the pelvis. Urethral injuries may present with blood at the meatus in which case a retrograde urethrogram is indicated. Blunt trauma accounts for approximately 90% of renal injuries and 1-5% of hospitalized trauma patients have a renal injury. In most cases, clinically significant renal injuries are associated with gross hematuria. Rarely, a significant injury is found with microscopic hematuria but also hemodynamic instability. Therefore, if renal injury is suspected, gross hematuria in the hemodynamically stablepatientdictates further imaging evaluation.
Incorrect
The presence of gross hematuria in the setting of trauma signifies a urologic injury until proven otherwise. Gross hematuria suggests either a lower or upper urinary tract injury. Bladder injury (e.g. rupture) is suggested by lower abdominal pain and tenderness or a positive FAST examination on evaluation of the pelvis. Urethral injuries may present with blood at the meatus in which case a retrograde urethrogram is indicated. Blunt trauma accounts for approximately 90% of renal injuries and 1-5% of hospitalized trauma patients have a renal injury. In most cases, clinically significant renal injuries are associated with gross hematuria. Rarely, a significant injury is found with microscopic hematuria but also hemodynamic instability. Therefore, if renal injury is suspected, gross hematuria in the hemodynamically stablepatientdictates further imaging evaluation.
Question 5 of 10
5. Question
Which of the following organs is the most commonly injured in adult blunt abdominal trauma?
Correct
The spleen is the most common organ injured in adults in blunt abdominal trauma. Most blunt abdominal trauma results from motor vehicle collisions. Many splenic injuries, including high-grade lacerations, may be managed conservatively in the hemodynamically stable patient without other significant intraabdominal injuries.
Incorrect
The spleen is the most common organ injured in adults in blunt abdominal trauma. Most blunt abdominal trauma results from motor vehicle collisions. Many splenic injuries, including high-grade lacerations, may be managed conservatively in the hemodynamically stable patient without other significant intraabdominal injuries.
Question 6 of 10
6. Question
When performing a FAST exam and assessing the spleno-renal (peri-splenic) window, which of the following is the most likely location for free fluid to accumulate?
Correct
The correct answer is between the spleen and the diaphragm. Fluid is rarely seen between the spleen and kidney. Most commonly it is between the spleen and diaphragm, or simply around the spleen.
Incorrect
The correct answer is between the spleen and the diaphragm. Fluid is rarely seen between the spleen and kidney. Most commonly it is between the spleen and diaphragm, or simply around the spleen.
Question 7 of 10
7. Question
A 32-year-old woman presents after a motor vehicle crash in which she was the restrained driver. She has abdominal pain and is hemodynamically stable. Abdominal examination reveals ecchymosis horizontally across the abdomen near the umbilicus. Bedside FAST examination results are negative, and a CT scan revels no injuries. Re-examination reveals a tender abdomen. What is the most appropriate management?
Correct
When a patient presents after suffering blunt abdominal trauma and has a concerning abdominal examination but a negative workup, decisions about appropriate management can be challenging. In this particular patient, the negative CT scan and FAST results are reassuring in terms of solid organ injury, but there is potential for other serious injuries, such as hollow viscus injury or pancreatic injury. The seatbelt sign (ecchymosis on the abdomen) suggests that the patient could have these injuries, so a period of observation in the hospital is warranted. Discharging the patient without a period of observation either in the hospital or in the emergency department creates the potential for the patient to suffer significant complications from undiagnosed intraabdominal injury. Diagnostic peritoneal lavage would be helpful in this patient if grossly positive but might not otherwise definitively determine the indication for surgical management. The formal abdominal ultrasound examination done in the radiology department is no more accurate than a FAST examination done by an experienced provider for identifying hollow viscus injury or acute pancreatic injury.
Incorrect
When a patient presents after suffering blunt abdominal trauma and has a concerning abdominal examination but a negative workup, decisions about appropriate management can be challenging. In this particular patient, the negative CT scan and FAST results are reassuring in terms of solid organ injury, but there is potential for other serious injuries, such as hollow viscus injury or pancreatic injury. The seatbelt sign (ecchymosis on the abdomen) suggests that the patient could have these injuries, so a period of observation in the hospital is warranted. Discharging the patient without a period of observation either in the hospital or in the emergency department creates the potential for the patient to suffer significant complications from undiagnosed intraabdominal injury. Diagnostic peritoneal lavage would be helpful in this patient if grossly positive but might not otherwise definitively determine the indication for surgical management. The formal abdominal ultrasound examination done in the radiology department is no more accurate than a FAST examination done by an experienced provider for identifying hollow viscus injury or acute pancreatic injury.
Question 8 of 10
8. Question
A 28-year-old woman presents after being involved in a high-speed motor vehicle collision. She is 32-weeks pregnant and was the back seat passenger wearing only a lap belt. Her blood pressure is 80/40 mm Hg, heart rate is 130 beats/minute, respiratory rate is 24 breaths/minute, temperature is 37°C, and oxygen saturation is 98% on 10 L via non-rebreather mask. Her abdomen is tender to palpation, with rebound and guarding. The fetus is felt in a transverse lie and you can easily palpate the fetal extremities. There is no fetal cardiac activity on ultrasound. What is the most likely diagnosis?
Correct
The patient is suffering from an acute traumatic uterine rupture a rare complication following blunt and penetrating abdominal trauma. The classic signs include maternal shock, abdominal pain, peritoneal signs, abnormal fetal lie, easily palpable fetal anatomy and fetal demise. The use of a lap belt alone increases the risk of developing uterine rupture and placental abruption. The patient should be aggressively resuscitated and emergent surgical and obstetric consultation is necessary for operative repair.
Incorrect
The patient is suffering from an acute traumatic uterine rupture a rare complication following blunt and penetrating abdominal trauma. The classic signs include maternal shock, abdominal pain, peritoneal signs, abnormal fetal lie, easily palpable fetal anatomy and fetal demise. The use of a lap belt alone increases the risk of developing uterine rupture and placental abruption. The patient should be aggressively resuscitated and emergent surgical and obstetric consultation is necessary for operative repair.
Question 9 of 10
9. Question
A 19-year-old male with no past medical history, presents with abdominal pain. He was skateboarding and doing tricks off a handrail, when he slipped and landed directly onto his abdomen against the handrail. He denies loss of consciousness, but has developed progressively worsening epigastric abdominal pain with nausea and vomiting. Vital signs are BP 124 /72, HR 99, RR 24, Temp 99.2°F, and 98% on room air. CT abdomen was negative. Electrolytes are within normal limits, WBC 13.1, Hemoglobin 14.5 g/dL, AST 56 IU/L, ALT 55 IU/L, Alkaline phosphatase 60 IU/L, total bilirubin 0.9 mg/dL, lipase 66 IU/L. Given the scenario, which of the following is the best next step for this patient?
Correct
CT imaging is excellent in detecting solid organ injury (e.g. kidney, liver, spleen, pancreas) but not good at detecting mesenteric hematoma/tears or hollow viscus injury.
Incorrect
CT imaging is excellent in detecting solid organ injury (e.g. kidney, liver, spleen, pancreas) but not good at detecting mesenteric hematoma/tears or hollow viscus injury.
Question 10 of 10
10. Question
What is the most common sign of bladder injury?
Correct
Gross hematuria is the most common sign of bladder injury and occurs in more than 95% of the cases. Bladder injury most commonly results from blunt motor vehicle trauma and 80% are associated with fracture of the bony pelvis. Bladder injuries include contusions, intraperitoneal and extraperitoneal ruptures and combination ruptures. Other signs of bladder rupture include suprapubic and lower abdominal pain, inability to urinate, pelvic fracture, and blood at the urethral meatus. Blood at the urethral meatus is a sign of underlying urethral or bladder injury, although is not the most common sign for bladder injury. A high-riding prostate is a sign of urethral injury, not bladder injury. An inability to urinate is a sign of bladder injury, however is not the most common sign.
Incorrect
Gross hematuria is the most common sign of bladder injury and occurs in more than 95% of the cases. Bladder injury most commonly results from blunt motor vehicle trauma and 80% are associated with fracture of the bony pelvis. Bladder injuries include contusions, intraperitoneal and extraperitoneal ruptures and combination ruptures. Other signs of bladder rupture include suprapubic and lower abdominal pain, inability to urinate, pelvic fracture, and blood at the urethral meatus. Blood at the urethral meatus is a sign of underlying urethral or bladder injury, although is not the most common sign for bladder injury. A high-riding prostate is a sign of urethral injury, not bladder injury. An inability to urinate is a sign of bladder injury, however is not the most common sign.
This week wraps up Trauma, covering Abdominal and Pelvic Trauma. We will begin with a quiz review, followed by oral boards with the Mamas and Papas, Dr. Aquino and Dr Twiner. We will then run FLIP with Drs. Vincent and McElroy.
You have already completed the quiz before. Hence you can not start it again.
Quiz is loading...
You must sign in or sign up to start the quiz.
You have to finish following quiz, to start this quiz:
Results
0 of 10 questions answered correctly
Your time:
Time has elapsed
You have reached 0 of 0 points, (0)
Average score
Your score
Categories
Orthopedic0%
1
2
3
4
5
6
7
8
9
10
Answered
Review
Question 1 of 10
1. Question
A 39-year-old woman is involved in a high-speed motor vehicle crash. There is significant intrusion of the front of the vehicle. X-rays show a posterior knee dislocation. Which of the following is a hard sign for vascular injury following knee dislocation?
Correct
A knee dislocation is a very serious injury which involves dislocation of the tibiofemoral joint and disruption of all the ligamentous structures. Posterior dislocations occur when with high velocity direct trauma to a flexed knee, such as a dashboard impact in a motor vehicle crash. The most serious consequence of knee dislocation is a popliteal artery injury. The popliteal artery runs in the popliteal fossa and its relative immobility makes it susceptible to disruption when dislocation occurs. A popliteal artery injury is a true emergency. When repair is delayed for more than 8 hours, the amputation rate is greater than 90%. Hard signs of vascular injury include absent pulses, limb ischemia (e.g. a cool, mottled foot), rapidly expanding hematoma, palpable thrill or audible bruit, or history of pulsatile bleeding (in the case of penetrating trauma or open injury). Though not considered a hard sign, paresthesias also raise concern for injury to popliteal structures. The workup-up for ruling out popliteal injury includes serial physical exams, ankle-brachial indices (ABI), CT angiography, conventional angiography, and duplex ultrasonography. Patients with vascular injury require emergent surgical repair.
Incorrect
A knee dislocation is a very serious injury which involves dislocation of the tibiofemoral joint and disruption of all the ligamentous structures. Posterior dislocations occur when with high velocity direct trauma to a flexed knee, such as a dashboard impact in a motor vehicle crash. The most serious consequence of knee dislocation is a popliteal artery injury. The popliteal artery runs in the popliteal fossa and its relative immobility makes it susceptible to disruption when dislocation occurs. A popliteal artery injury is a true emergency. When repair is delayed for more than 8 hours, the amputation rate is greater than 90%. Hard signs of vascular injury include absent pulses, limb ischemia (e.g. a cool, mottled foot), rapidly expanding hematoma, palpable thrill or audible bruit, or history of pulsatile bleeding (in the case of penetrating trauma or open injury). Though not considered a hard sign, paresthesias also raise concern for injury to popliteal structures. The workup-up for ruling out popliteal injury includes serial physical exams, ankle-brachial indices (ABI), CT angiography, conventional angiography, and duplex ultrasonography. Patients with vascular injury require emergent surgical repair.
Question 2 of 10
2. Question
Which of the following statements regarding carpal tunnel syndrome is correct?
Correct
Carpal tunnel syndrome is a median mononeuropathy and the most common mononeuropathy in the body. Classic symptoms include pain and paresthesias on the volar side of the first, second, and third fingers and radial half of the fourth finger with symptoms most notable at night. Fourth finger sensory dysfunction on the radial aspect of the finger only is very specific for the diagnosis of carpal tunnel syndrome. Extension and flexion at the wrist commonly exacerbate the condition and can aid in diagnosis. Using the Phalen test, by holding the wrists in maximum flexion for at least 1 minute, reproduction of the symptoms helps to point toward this diagnosis. A positive Tinel sign yields tingling after tapping on the wrist where the median nerve passes through the carpal tunnel between the carpal bones and the flexor retinaculum. Both of these tests have poor sensitivity, and if negative, do not effectively rule out the diagnosis. A more specific finding is divided sensation of the fourth finger, with dysfunction on the radial aspect only. Treatment for carpal tunnel syndrome includes a splint on the wrist and NSAIDs. Recurrent symptoms sometimes warrant steroid injections by a hand surgeon and possibly surgical decompression.
WHY THE OTHER ANSWERS ARE WRONG
Some patients have weakness in the muscles of the hand, a condition known as Guyon canal syndrome or handlebar palsy. It is an ulnar mononeuropathy rather than a median neuropathy as with carpal tunnel syndrome. In this rare syndrome, there is compression of the ulnar nerve at the wrist that causes weakness of the intrinsic muscles of the hand. What often happens is that the sensory function of the ulnar nerve is spared or involves only the palmar aspect of the fifth finger and half of the fourth. Carpal tunnel can cause weakness in the thenar eminence, but not the traditional intrinsic muscles of the hand.
Carpal tunnel syndrome is caused by compression of the medal nerve in the inflamed carpal tunnel between the flexor retinaculum and carpal bones, not at the medial epicondyle. Compression here causes cubital tunnel syndrome, and patients have numbness and tingling in the fifth and lateral fourth digits.
Carpal tunnel syndrome is the most common mononeuropathy in the body and involves the median nerve, not the ulnar nerve. The most common ulnar nerve mononeuropathy is cubital tunnel syndrome. Patients typically have paresthesias of the fifth and lateral fourth fingers. Ultimately, weakness of the intrinsic muscles can ensue.
Incorrect
Carpal tunnel syndrome is a median mononeuropathy and the most common mononeuropathy in the body. Classic symptoms include pain and paresthesias on the volar side of the first, second, and third fingers and radial half of the fourth finger with symptoms most notable at night. Fourth finger sensory dysfunction on the radial aspect of the finger only is very specific for the diagnosis of carpal tunnel syndrome. Extension and flexion at the wrist commonly exacerbate the condition and can aid in diagnosis. Using the Phalen test, by holding the wrists in maximum flexion for at least 1 minute, reproduction of the symptoms helps to point toward this diagnosis. A positive Tinel sign yields tingling after tapping on the wrist where the median nerve passes through the carpal tunnel between the carpal bones and the flexor retinaculum. Both of these tests have poor sensitivity, and if negative, do not effectively rule out the diagnosis. A more specific finding is divided sensation of the fourth finger, with dysfunction on the radial aspect only. Treatment for carpal tunnel syndrome includes a splint on the wrist and NSAIDs. Recurrent symptoms sometimes warrant steroid injections by a hand surgeon and possibly surgical decompression.
WHY THE OTHER ANSWERS ARE WRONG
Some patients have weakness in the muscles of the hand, a condition known as Guyon canal syndrome or handlebar palsy. It is an ulnar mononeuropathy rather than a median neuropathy as with carpal tunnel syndrome. In this rare syndrome, there is compression of the ulnar nerve at the wrist that causes weakness of the intrinsic muscles of the hand. What often happens is that the sensory function of the ulnar nerve is spared or involves only the palmar aspect of the fifth finger and half of the fourth. Carpal tunnel can cause weakness in the thenar eminence, but not the traditional intrinsic muscles of the hand.
Carpal tunnel syndrome is caused by compression of the medal nerve in the inflamed carpal tunnel between the flexor retinaculum and carpal bones, not at the medial epicondyle. Compression here causes cubital tunnel syndrome, and patients have numbness and tingling in the fifth and lateral fourth digits.
Carpal tunnel syndrome is the most common mononeuropathy in the body and involves the median nerve, not the ulnar nerve. The most common ulnar nerve mononeuropathy is cubital tunnel syndrome. Patients typically have paresthesias of the fifth and lateral fourth fingers. Ultimately, weakness of the intrinsic muscles can ensue.
Question 3 of 10
3. Question
According to the Ottawa Knee Rules, which of the following is NOT an indication for knee x-ray?
Correct
The correct answer is inability to fully extend the knee. The remaining answer choices are all components of the Ottawa Knee Rules, which include: Age over 55 years old, isolated patella tenderness, tenderness at head of fibula, inability to flex knee 90 degrees, and inability to bear weight (4 steps) immediately after injury or in the ED.
Incorrect
The correct answer is inability to fully extend the knee. The remaining answer choices are all components of the Ottawa Knee Rules, which include: Age over 55 years old, isolated patella tenderness, tenderness at head of fibula, inability to flex knee 90 degrees, and inability to bear weight (4 steps) immediately after injury or in the ED.
Question 4 of 10
4. Question
A 23 year old female is brought in by ambulance to the emergency department after a fall off of a horse. She has an isolated right femur fracture. The extremity is reduced and splinted. While awaiting further imaging studies, she develops severe pain. The splint is removed, and on physical exam the patient is found to have a tense, swollen and extremely tender right thigh. Which of the following findings is suggestive of compartment syndrome?
Correct
Compartment syndrome is diagnosed by a compartment pressure greater than 30 or a ∆ measurement (diastolic BP – compartment pressure) <30. This patient's pressure difference is 25 which is strongly suggestive of compartment syndrome. The femur is in a large compartment and it is typically difficult to sustain compartment syndrome. More concerning areas are the anterior tibial compartment and the forearm.
Incorrect
Compartment syndrome is diagnosed by a compartment pressure greater than 30 or a ∆ measurement (diastolic BP – compartment pressure) <30. This patient's pressure difference is 25 which is strongly suggestive of compartment syndrome. The femur is in a large compartment and it is typically difficult to sustain compartment syndrome. More concerning areas are the anterior tibial compartment and the forearm.
Question 5 of 10
5. Question
A 25-year-old woman comes to the emergency department because of right ankle pain sustained by twisting her ankle while playing volleyball six hours ago. She was able to bear partial weight immediately after the injury. She has no medical problems and takes no medications. On physical examination she is still able to bear partial weight on her right leg but demonstrates an antalgic gait. There is mild edema around the lateral malleolus. Squeeze test of the lower leg is negative. Neurovascular examination of the right leg shows no abnormalities. X-ray study is shown. Which of the following ligaments is most likely to be injured in this patient’s condition?
A. Anterior talofibular ligament
The correct answer is anterior talofibular ligament (ATFL) injury. The anterior-posterior and oblique x-ray views of the right ankle demonstrate a normal mortise and no fractures. There is, however, right lateral ankle soft tissue swelling. Typically, the mechanism of injury helps determine the location of ligamentous injury. The most common mechanism is inversion of the plantar-flexed foot, leading to lateral ankle sprains due to injury of the anterior talofibular ligament (ATFL), the calcaneofibular ligament, and posterior talofibular ligament. Of the three ligaments, the ATFL is the most commonly injured.
B. Calcaneofibular ligament
This patient is presenting with an ankle sprain and with lateral ankle sprains, the most common ligament injured is the anterior talofibular ligament. The calcaneofibular ligament may also be injured, but isolated calcaneofibular ligament injury is uncommon.
C. Deltoid ligament
This patient is presenting with a lateral ankle sprain and the deltoid ligament is on the medial side of the ankle, is much stronger, and is much less prone to injury.
D. Tibiofibular syndesmosis
Injury to the tibiofibular syndesmosis is an uncommon, but significant, injury. Injury to this structure characterizes a “high” ankle sprain. However, the squeeze test of the tibia-fibula is reported negative, making this diagnosis less likely. The squeeze test involves compression of the fibula against the tibia at the mid-calf level, trying to reproduce pain while attempting to widen the syndesmosis between the distal tibiofibular space.
Incorrect
A. Anterior talofibular ligament
The correct answer is anterior talofibular ligament (ATFL) injury. The anterior-posterior and oblique x-ray views of the right ankle demonstrate a normal mortise and no fractures. There is, however, right lateral ankle soft tissue swelling. Typically, the mechanism of injury helps determine the location of ligamentous injury. The most common mechanism is inversion of the plantar-flexed foot, leading to lateral ankle sprains due to injury of the anterior talofibular ligament (ATFL), the calcaneofibular ligament, and posterior talofibular ligament. Of the three ligaments, the ATFL is the most commonly injured.
B. Calcaneofibular ligament
This patient is presenting with an ankle sprain and with lateral ankle sprains, the most common ligament injured is the anterior talofibular ligament. The calcaneofibular ligament may also be injured, but isolated calcaneofibular ligament injury is uncommon.
C. Deltoid ligament
This patient is presenting with a lateral ankle sprain and the deltoid ligament is on the medial side of the ankle, is much stronger, and is much less prone to injury.
D. Tibiofibular syndesmosis
Injury to the tibiofibular syndesmosis is an uncommon, but significant, injury. Injury to this structure characterizes a “high” ankle sprain. However, the squeeze test of the tibia-fibula is reported negative, making this diagnosis less likely. The squeeze test involves compression of the fibula against the tibia at the mid-calf level, trying to reproduce pain while attempting to widen the syndesmosis between the distal tibiofibular space.
Question 6 of 10
6. Question
A 25-year-old man involved in a motor vehicle collision presents with left knee pain. On examination, the knee is diffusely swollen with limited range of motion. An X-ray is shown. A sensory deficit in which of the following locations is associated with this injury?
Correct
The X-ray demonstrates an anterior knee dislocation. It is important to differentiate between a knee dislocation (tibiofemoral) and patellar dislocation. The patellar dislocation is less severe and typically without other associated injuries. A knee dislocation may not always be present on arrival to the ED as spontaneous reduction can occur prior to arrival. However, even with spontaneous reduction, significant pain and soft tissue swelling remains. In an anterior knee dislocation surrounding vascular and nerve structures are at risk. The dislocation results in significant ligamentous injury. The neurovascular bundle running behind the knee is at risk. The life threatening injury is laceration of the popliteal artery within this bundle. In addition to lacerations of the vascular structure, dissection is also possible from the shear injury occurring during the trauma. The popliteal vein is the other vascular structure within the bundle. The common peroneal nerve is located within the bundle and the neurologic injury associated with an anterior dislocation. This dislocation is associated with an injury to the common peroneal nerve in 20 to 40% of patients. The common peroneal nerve arises from the L4-S2 nerve roots and branches into the deep and superficial peroneal nerves. These nerves supply sensation to most of the dorsum of the foot sparing the lateral most portion of the foot. The motor function of the common peroneal is dorsiflexion of the ankle.
Incorrect
The X-ray demonstrates an anterior knee dislocation. It is important to differentiate between a knee dislocation (tibiofemoral) and patellar dislocation. The patellar dislocation is less severe and typically without other associated injuries. A knee dislocation may not always be present on arrival to the ED as spontaneous reduction can occur prior to arrival. However, even with spontaneous reduction, significant pain and soft tissue swelling remains. In an anterior knee dislocation surrounding vascular and nerve structures are at risk. The dislocation results in significant ligamentous injury. The neurovascular bundle running behind the knee is at risk. The life threatening injury is laceration of the popliteal artery within this bundle. In addition to lacerations of the vascular structure, dissection is also possible from the shear injury occurring during the trauma. The popliteal vein is the other vascular structure within the bundle. The common peroneal nerve is located within the bundle and the neurologic injury associated with an anterior dislocation. This dislocation is associated with an injury to the common peroneal nerve in 20 to 40% of patients. The common peroneal nerve arises from the L4-S2 nerve roots and branches into the deep and superficial peroneal nerves. These nerves supply sensation to most of the dorsum of the foot sparing the lateral most portion of the foot. The motor function of the common peroneal is dorsiflexion of the ankle.
Question 7 of 10
7. Question
A 19-year-old man presents after twisting his ankle. When stepping off a curb, he inverted his ankle and now has pain on the lateral malleolus and along the side of the foot. Which of the following is the most appropriate management of the injury shown in the X-ray?
Correct
Metatarsal fractures are responsible for one third of foot fractures. Fractures at the base of the metatarsal are uncommon in the first through fourth metatarsals. The isolated fracture at the base of the fifth metatarsal is common and management depends on which form of the two fractures occurs. The Jones fracture is a transverse fracture occurring in the diaphysis of the bone at least 1.5 cm distal to the end of the bone. The Jones fracture tends to displace with further weight bearing. With improper immobilization there is a higher percentage of nonunion of the fracture fragments. Therefore, patients should be immobilized in a posterior splint and given crutches for non-weight bearing ambulation. The pseudo-Jones fracture is an avulsion fracture of the tuberosity at the base of the fifth metatarsal. The injury often occurs as a result of a lateral ankle strain with tension at the attachment of the peroneus brevis tendon. These patients are treated with a compression dressing and weight bearing as tolerated. Providers may elect to give the patients a hard sole shoe.
Incorrect
Metatarsal fractures are responsible for one third of foot fractures. Fractures at the base of the metatarsal are uncommon in the first through fourth metatarsals. The isolated fracture at the base of the fifth metatarsal is common and management depends on which form of the two fractures occurs. The Jones fracture is a transverse fracture occurring in the diaphysis of the bone at least 1.5 cm distal to the end of the bone. The Jones fracture tends to displace with further weight bearing. With improper immobilization there is a higher percentage of nonunion of the fracture fragments. Therefore, patients should be immobilized in a posterior splint and given crutches for non-weight bearing ambulation. The pseudo-Jones fracture is an avulsion fracture of the tuberosity at the base of the fifth metatarsal. The injury often occurs as a result of a lateral ankle strain with tension at the attachment of the peroneus brevis tendon. These patients are treated with a compression dressing and weight bearing as tolerated. Providers may elect to give the patients a hard sole shoe.
Question 8 of 10
8. Question
A 36-year-old woman presents with right wrist pain. She tripped over a step and braced herself with her right hand as she fell forward. A lateral radiographs of her right wrist is shown. What is the most likely diagnosis?
Correct
The lateral radiograph shows a perilunate dislocation in which the lunate remains aligned with the distal radius, however the remaining carpal bones are dorsally displaced. There is significant ligamentous injury. The mechanism of injury is generally a fall on an outstretched hand (FOOSH). Patients complain of wrist pain and swelling, and tenderness is noted over the carpal bones on exam. If untreated, the patient is at risk for developing median nerve compression, avascular necrosis, compartment syndrome and long-term disability. A hand surgeon should promptly evaluate the patient with a plan for urgent open dislocation reduction.
Incorrect
The lateral radiograph shows a perilunate dislocation in which the lunate remains aligned with the distal radius, however the remaining carpal bones are dorsally displaced. There is significant ligamentous injury. The mechanism of injury is generally a fall on an outstretched hand (FOOSH). Patients complain of wrist pain and swelling, and tenderness is noted over the carpal bones on exam. If untreated, the patient is at risk for developing median nerve compression, avascular necrosis, compartment syndrome and long-term disability. A hand surgeon should promptly evaluate the patient with a plan for urgent open dislocation reduction.
Question 9 of 10
9. Question
A 24-year-old man presents with shoulder pain after reaching up to change a light bulb. On examination, deformity is noted to the right shoulder. His X-ray is shown. What is the most commonly associated injury?
Correct
Anterior shoulder dislocations are the most common type of shoulder dislocation. The axillary nerve is the neurologic structure most at risk for injury during a shoulder dislocation. Sensation over the deltoid muscle is the way in which axillary nerve function is tested on physical examination. Additionally, distal nerve function of the median, radial and ulnar nerves should be evaluated as part of the complete evaluation of the shoulder joint. It is important to document all findings of the neurovascular examination prior to any attempts as a baseline. Posterior dislocations are associated with significant injury 30% of the time to intrathoracic and mediastinal structures. These complications include: injury to the great vessels, tracheoesophageal fistula, pneumothorax and brachial plexus injuries.
Incorrect
Anterior shoulder dislocations are the most common type of shoulder dislocation. The axillary nerve is the neurologic structure most at risk for injury during a shoulder dislocation. Sensation over the deltoid muscle is the way in which axillary nerve function is tested on physical examination. Additionally, distal nerve function of the median, radial and ulnar nerves should be evaluated as part of the complete evaluation of the shoulder joint. It is important to document all findings of the neurovascular examination prior to any attempts as a baseline. Posterior dislocations are associated with significant injury 30% of the time to intrathoracic and mediastinal structures. These complications include: injury to the great vessels, tracheoesophageal fistula, pneumothorax and brachial plexus injuries.
Question 10 of 10
10. Question
A 25 year old presents with left ankle pain. She reports twisting her ankle while running in a field. She was able to walk immediately after the injury. She has no medical problems and takes no medications. On examination she is able to weight bear, but only walks on her toes on the left side. There is a small amount of ankle swelling to both malleoli on the left side. Her foot is neurovascularly intact. There is no bony tendernes on the posterior edges of the medial or lateral malleoli. The patient has pain with compression of the fibula and tibia at the mid-calf level as well as when she crosses her left leg on her right knee. The x-ray image is shown. What is the best management plan?
Correct
This case describes a patient with an ankle syndesmosis injury which if not properly treated leads to chronic ankle instability and pain. The image shows widening of the tibio-fibular joint as well as a widened ankle mortise. This ankle requires non-weight bearing an urgent othopedic consultation for possible operative fixation.
Incorrect
This case describes a patient with an ankle syndesmosis injury which if not properly treated leads to chronic ankle instability and pain. The image shows widening of the tibio-fibular joint as well as a widened ankle mortise. This ankle requires non-weight bearing an urgent othopedic consultation for possible operative fixation.
This week we continue the Trauma Train with orthopedic injuries. We will begin conference with the good Doctors Mayhem Melhelm and Dikeman…. Dikeman. This will be followed by Journal Club, and then a resident applicant input meeting to help out with the rank list. There will be food.
This weeks content covers a ton of small topics and injuries. Harwood & Nuss or HIPPOEM (both linked below) are the best comprehensive source to cover your bases. You will otherwise be missing out on a lot.
HIPPOEM — If you have access (interns should!), go through all of the HIPPO EM MSK lectures, probably the best succinct review on all need-to-know fractures outside of the text
You have already completed the quiz before. Hence you can not start it again.
Quiz is loading...
You must sign in or sign up to start the quiz.
You have to finish following quiz, to start this quiz:
Results
0 of 10 questions answered correctly
Your time:
Time has elapsed
You have reached 0 of 0 points, (0)
Average score
Your score
Categories
Trauma0%
1
2
3
4
5
6
7
8
9
10
Answered
Review
Question 1 of 10
1. Question
When performing the emergency department thoracotomy after the incision has been made and the pleural cavity has been entered in the presence of cardiac arrest and no obvious injury is seen on entry what is the next step that should be taken?
Correct
It is difficult to identify cardiac tamponade by visual inspection only. Thus the first step should be opening the pericardium. Direct cardiac massage should be done after pericardium is opened and if no tamponade is found. Cross clamping of the aorta should be the next step if the patient still has no cardiac activity.
Incorrect
It is difficult to identify cardiac tamponade by visual inspection only. Thus the first step should be opening the pericardium. Direct cardiac massage should be done after pericardium is opened and if no tamponade is found. Cross clamping of the aorta should be the next step if the patient still has no cardiac activity.
Question 2 of 10
2. Question
An 18-year-old man presents after a motor vehicle collision (MVC) in which he was ejected from the vehicle. The paramedics have been administering bag-valve-mask ventilation en route because of respiratory distress and now report increased resistance with ventilations. The patient has decreased breath sounds on the left. His blood pressure is 80/40 mm Hg, and his pulse is 145 beats per minute. His respirations are agonal, with a rate of 5 breaths per minute. Which of the following is the most appropriate next step in the management of this patient?
Correct
This clinical scenario depicts a patient with a tension pneumothorax. He has decreased blood pressure, decreased breath sounds, and, most important, an increased resistance to ventilation, which is the earliest sign of the development of a tension pneumothorax. Immediate decompression with a large-bore needle is the correct initial management in this condition.
Incorrect
This clinical scenario depicts a patient with a tension pneumothorax. He has decreased blood pressure, decreased breath sounds, and, most important, an increased resistance to ventilation, which is the earliest sign of the development of a tension pneumothorax. Immediate decompression with a large-bore needle is the correct initial management in this condition.
Question 3 of 10
3. Question
Which of the following chest radiographic findings suggests underlying traumatic aortic disruption?
Correct
Elevation of the right mainstem bronchus is suggestive of acute traumatic aortic disruption. Traumatic aortic disruption can result in immediate death if the degree of injury is severe, such as a complete transection. However, with less severe disruptions, the patient may survive and present to the ED. Prompt diagnosis and early intervention, can lead to a functional recovery.
Deviation of the trachea to the right (B), not the left suggests aortic injury. Deviation of the esophagus to the right (A), not the left suggests aortic injury. A left-sided hemothorax (D) suggests aortic injury, not a right hemothorax.
Two most common sites of aortic injury with blunt trauma are: Aortic isthmus and ascending aorta just proximal to the origin of the brachiocepahic vessels.
Incorrect
Elevation of the right mainstem bronchus is suggestive of acute traumatic aortic disruption. Traumatic aortic disruption can result in immediate death if the degree of injury is severe, such as a complete transection. However, with less severe disruptions, the patient may survive and present to the ED. Prompt diagnosis and early intervention, can lead to a functional recovery.
Deviation of the trachea to the right (B), not the left suggests aortic injury. Deviation of the esophagus to the right (A), not the left suggests aortic injury. A left-sided hemothorax (D) suggests aortic injury, not a right hemothorax.
Two most common sites of aortic injury with blunt trauma are: Aortic isthmus and ascending aorta just proximal to the origin of the brachiocepahic vessels.
Question 4 of 10
4. Question
A 23-year-old man presents to the ED with chest pain after a head-on high-speed motor vehicle collision. Vital signs are BP 115/65 mm Hg, HR 130 beats/minute, RR 22 breaths/minute, and T 98.4°F. On auscultation of the chest, you hear a friction rub. His chest X-ray reveals a sternal fracture. An ECG reveals anterior T wave inversions and frequent premature ventricular complexes. Which of the following structures is most commonly injured in this disease process?
Correct
This patient has signs and symptoms of blunt cardiac injury. The most common chamber injured is the right ventricle as it lies most anterior in the thorax, immediately behind the sternum. Blunt cardiac injury results in myocardial contusion and “stunning” of the heart muscle. This can lead to dysrhythmias and heart failure. Cardiac contusion is most frequently caused by high-speed motor vehicle collisions or severe crush injuries. The most common etiology is a depressed sternal fracture. Symptoms of cardiac contusion include chest pain, shortness of breath, and palpitations. There is often external evidence of chest trauma including obvious depressed fractures of the sternum and steering wheel imprints. The diagnosis of significant blunt cardiac injury is made by electrocardiogram abnormalities (e.g. T wave inversions and frequent PVCs) and a serum troponin level. If either of these studies is abnormal, the patient should be admitted for telemetry monitoring.
Incorrect
This patient has signs and symptoms of blunt cardiac injury. The most common chamber injured is the right ventricle as it lies most anterior in the thorax, immediately behind the sternum. Blunt cardiac injury results in myocardial contusion and “stunning” of the heart muscle. This can lead to dysrhythmias and heart failure. Cardiac contusion is most frequently caused by high-speed motor vehicle collisions or severe crush injuries. The most common etiology is a depressed sternal fracture. Symptoms of cardiac contusion include chest pain, shortness of breath, and palpitations. There is often external evidence of chest trauma including obvious depressed fractures of the sternum and steering wheel imprints. The diagnosis of significant blunt cardiac injury is made by electrocardiogram abnormalities (e.g. T wave inversions and frequent PVCs) and a serum troponin level. If either of these studies is abnormal, the patient should be admitted for telemetry monitoring.
Question 5 of 10
5. Question
A 25-year-old football player is brought in to the emergency department with neck pain and paresthesias after tackling another player head-on. Physical exam is significant for cervical spine tenderness to palpation. CT is not available, so spinal precautions are maintained and plain films are obtained. What type of fracture pattern is seen in the associated xray?
Correct
This xray shows a Jefferson fracture, or a C1 burst fracture. These fractures occur with axial loading with vertical compression, an injury pattern that is common when a football player spears another player. When looking at the odontoid view of a c-spine film, the lateral masses of C1 shoulder align with the lateral parts of the C2 vertebral body. A Hangman’s fracture, or bilateral C2 pedicle fracture is best seen on a lateral view and shows C2 displaced anteriorly on C3. Odontoid fractures involve the odontoid process and are graded type I through type III depending on position. Type II and III are both unstable fractures. Bilateral facet dislocation fractures are seen on lateral xrays of the c-spine and is characterized by anterior displacement greater than 50% diameter of vertebral body.
Incorrect
This xray shows a Jefferson fracture, or a C1 burst fracture. These fractures occur with axial loading with vertical compression, an injury pattern that is common when a football player spears another player. When looking at the odontoid view of a c-spine film, the lateral masses of C1 shoulder align with the lateral parts of the C2 vertebral body. A Hangman’s fracture, or bilateral C2 pedicle fracture is best seen on a lateral view and shows C2 displaced anteriorly on C3. Odontoid fractures involve the odontoid process and are graded type I through type III depending on position. Type II and III are both unstable fractures. Bilateral facet dislocation fractures are seen on lateral xrays of the c-spine and is characterized by anterior displacement greater than 50% diameter of vertebral body.
Question 6 of 10
6. Question
A 45-year old male is brought in by EMS after being found down. Physical exam is significant for ecchymosis around the left orbit, multiple teeth avulsions, as well as C-spine tenderness to palpation. C-spine xray is shown below. Which of the following is correct regarding this patient’s injuries?
Correct
The correct answer is that this injury is associated with anterior cord syndrome. This patient has a flexion teardrop injury at C5 (note the wedge shaped fracture at the base of the vertebral body). This is generally caused by extreme flexion and axial load, classically a diving injury. There is failure of the anterior column, often middle column, and disruption of the posterior ligamentous complex and the significant potential for cord injury from retropulsion of a fragment as a result.
Incorrect
The correct answer is that this injury is associated with anterior cord syndrome. This patient has a flexion teardrop injury at C5 (note the wedge shaped fracture at the base of the vertebral body). This is generally caused by extreme flexion and axial load, classically a diving injury. There is failure of the anterior column, often middle column, and disruption of the posterior ligamentous complex and the significant potential for cord injury from retropulsion of a fragment as a result.
Question 7 of 10
7. Question
A 23-year-old male is involved in a high speed motor vehicle accident. He has multiple injuries, including a fracture at the junction of the odontoid and body of C2. What type of fracture does this patient have?
Correct
Odontoid (dens) fractures are classified as Type I, II and III. Type I fractures are stable avulsion injuries to the tip of the odontoid. This patient has a Type II fracture, which is a fracture at the junction of the odontoid and body of C2. A Type III fracture is a fracture at the base of the the dens. Both Type II and III fractures are unstable because of ligamentous attachments. A bilateral C2 pedicle fracture is also known as a Hangman’s fracture and is seen on lateral xray as C2 being displaced anteriorly on C3. A teardrop fracture is an avulsion fracture of the anteroinferior portion of the vertebral body. Both of the former fractures patterns are unstable c-spine injuries.
Incorrect
Odontoid (dens) fractures are classified as Type I, II and III. Type I fractures are stable avulsion injuries to the tip of the odontoid. This patient has a Type II fracture, which is a fracture at the junction of the odontoid and body of C2. A Type III fracture is a fracture at the base of the the dens. Both Type II and III fractures are unstable because of ligamentous attachments. A bilateral C2 pedicle fracture is also known as a Hangman’s fracture and is seen on lateral xray as C2 being displaced anteriorly on C3. A teardrop fracture is an avulsion fracture of the anteroinferior portion of the vertebral body. Both of the former fractures patterns are unstable c-spine injuries.
Question 8 of 10
8. Question
Which of the following is considered a stable cervical spine injury?
Correct
A Clay-shoveler’s fracture, which is an avulsion of the spinous process of C6 or C7, is considered a stable cervical spine fracture. Other stable injuries include a unilateral facet dislocation and a type I odontoid fracture in which the tip of the odontoid is broken off is usually a stable cervical spine injury. This injury got its name from clay shovelers who were shoveling heavy clay that caused them to abruptly flex their neck while contracting their lower neck muscles. Now, most cases are due to direct trauma to the posterior neck.
Incorrect
A Clay-shoveler’s fracture, which is an avulsion of the spinous process of C6 or C7, is considered a stable cervical spine fracture. Other stable injuries include a unilateral facet dislocation and a type I odontoid fracture in which the tip of the odontoid is broken off is usually a stable cervical spine injury. This injury got its name from clay shovelers who were shoveling heavy clay that caused them to abruptly flex their neck while contracting their lower neck muscles. Now, most cases are due to direct trauma to the posterior neck.
Question 9 of 10
9. Question
Which of the following trauma patients meets National Emergency X-Radiography Utilization Study (NEXUS) criteria for avoidance of cervical spine imaging?
Correct
Midline posterior bony cervical-spine tenderness is present if the patient reports pain on palpation of the posterior midline neck from the nuchal ridge to the prominence of the first thoracic vertebra, or if pain can be elicited with direct palpation of any cervical spinal process. Patients are considered intoxicated if any of the following is present: a recent history provided by the patient or an observer of intoxication or ingestion of an intoxicating substance, evidence of intoxication on physical exam (e.g., slurred speech, ataxia, dysmetria, or other cerebellar findings), or any behavior consistent with intoxication. Patients may also be considered intoxicated if tests of bodily fluids are positive for alcohol or drugs that affect the level of alertness. An altered level of consciousness can include any of the following: a GCS of 14 or less; disorientation to person, place, time, or events; an inability to remember three objects at five minutes; or a delayed or inappropriate response to external stimuli. A focal neurologic deficit is defined by any neurologic finding on motor or sensory exam. No precise definition of painful or distracting injury is possible. Determination of this is left to the treating clinician.Therefore, only the 64-year-old man with no posterior midline cervical spine tenderness nor pain when he turns his head to the left can be clinically cleared.
A 22-year-old man with no posterior midline cervical spine tenderness and the odor of alcohol on his breath (A), a 32-year-old man with no posterior midline cervical spine tenderness with a GCS of 14 (B) and an 18-year-old woman with no posterior midline cervical spine tenderness and mild numbness to her right arm (D) do not meet NEXUS criteria and require imaging.
What 3 components of the Canadian cervical spine rules are not present in the NEXUS cervical spine rules? Age, mechanism, neck rotational
Incorrect
Midline posterior bony cervical-spine tenderness is present if the patient reports pain on palpation of the posterior midline neck from the nuchal ridge to the prominence of the first thoracic vertebra, or if pain can be elicited with direct palpation of any cervical spinal process. Patients are considered intoxicated if any of the following is present: a recent history provided by the patient or an observer of intoxication or ingestion of an intoxicating substance, evidence of intoxication on physical exam (e.g., slurred speech, ataxia, dysmetria, or other cerebellar findings), or any behavior consistent with intoxication. Patients may also be considered intoxicated if tests of bodily fluids are positive for alcohol or drugs that affect the level of alertness. An altered level of consciousness can include any of the following: a GCS of 14 or less; disorientation to person, place, time, or events; an inability to remember three objects at five minutes; or a delayed or inappropriate response to external stimuli. A focal neurologic deficit is defined by any neurologic finding on motor or sensory exam. No precise definition of painful or distracting injury is possible. Determination of this is left to the treating clinician.Therefore, only the 64-year-old man with no posterior midline cervical spine tenderness nor pain when he turns his head to the left can be clinically cleared.
A 22-year-old man with no posterior midline cervical spine tenderness and the odor of alcohol on his breath (A), a 32-year-old man with no posterior midline cervical spine tenderness with a GCS of 14 (B) and an 18-year-old woman with no posterior midline cervical spine tenderness and mild numbness to her right arm (D) do not meet NEXUS criteria and require imaging.
What 3 components of the Canadian cervical spine rules are not present in the NEXUS cervical spine rules? Age, mechanism, neck rotational
Question 10 of 10
10. Question
A 66-year old woman is a restrained driver in a motor vehicle collision who presents to the ED with chest pain and an oxygen saturation of 93% on 2 L nasal cannula. She has ecchymosis and tenderness over her right thorax. Breath sounds are equal. Which of the following is true regarding her condition?
Correct
A pulmonary contusion occurs most commonly from blunt trauma and is the result of a direct bruise to the lung parenchyma followed by alveolar edema and hemorrhage. Although the initial radiographic findings (patchy, irregular to dense pulmonary infiltrates over the injured area) may be non-diagnostic and initial measurements of gaseous exchange may be normal, pulmonary function may be compromised over the ensuing few hours. Subsequent radiographs taken over several hours may demonstrate irregular opacification of the pulmonary parenchyma in a non-lobular pattern. Chest CT scans may demonstrate a contusion. Uncomplicated pulmonary contusions typically develop over the first 24 hours and resolve in approximately one week. Patients with severe pulmonary contusions experience difficulty breathing and hypoxia. Repeat chest radiographs show an increasing opacity in the affected lung fields. The condition is exacerbated if extensive crystalloid fluid resuscitation has been performed. Blood gas evaluation reveals an increased a-A gradient. Treatment is initially supplemental oxygen administration to reverse hypoxia; however, subsequent intubation and ventilation with positive end-expiratory pressure may be required. Other adjunctive therapies include pulmonary toilet and analgesia.
Air within the pleural cavity (A), or pneumothorax, is part of the differential diagnosis for this patient but is less likely to occur than a pulmonary contusion with a blunt trauma mechanism. Pulmonary contusions tend to worsen over time (B) and the patient’s initial presentation can change rapidly as there is an increase in alveolar edema and hemorrhage. Lung involvement is usually localized to a segment or a lobe (D). Adult respiratory distress syndrome (ARDS) is associated with diffuse lung involvement.
Incorrect
A pulmonary contusion occurs most commonly from blunt trauma and is the result of a direct bruise to the lung parenchyma followed by alveolar edema and hemorrhage. Although the initial radiographic findings (patchy, irregular to dense pulmonary infiltrates over the injured area) may be non-diagnostic and initial measurements of gaseous exchange may be normal, pulmonary function may be compromised over the ensuing few hours. Subsequent radiographs taken over several hours may demonstrate irregular opacification of the pulmonary parenchyma in a non-lobular pattern. Chest CT scans may demonstrate a contusion. Uncomplicated pulmonary contusions typically develop over the first 24 hours and resolve in approximately one week. Patients with severe pulmonary contusions experience difficulty breathing and hypoxia. Repeat chest radiographs show an increasing opacity in the affected lung fields. The condition is exacerbated if extensive crystalloid fluid resuscitation has been performed. Blood gas evaluation reveals an increased a-A gradient. Treatment is initially supplemental oxygen administration to reverse hypoxia; however, subsequent intubation and ventilation with positive end-expiratory pressure may be required. Other adjunctive therapies include pulmonary toilet and analgesia.
Air within the pleural cavity (A), or pneumothorax, is part of the differential diagnosis for this patient but is less likely to occur than a pulmonary contusion with a blunt trauma mechanism. Pulmonary contusions tend to worsen over time (B) and the patient’s initial presentation can change rapidly as there is an increase in alveolar edema and hemorrhage. Lung involvement is usually localized to a segment or a lobe (D). Adult respiratory distress syndrome (ARDS) is associated with diffuse lung involvement.
Trauma Week 2! This week we cover all trauma spinal and thoracic. We will begin with a quiz review, followed by oral boards by the human golden labrador, Dr. Farley, and human woof-dawg Dr. Rooney. After FLIP, created by Dr’s Liu and Wong, we will have M&M by Dr. Min-Venditti.
You have already completed the quiz before. Hence you can not start it again.
Quiz is loading...
You must sign in or sign up to start the quiz.
You have to finish following quiz, to start this quiz:
Results
0 of 10 questions answered correctly
Your time:
Time has elapsed
You have reached 0 of 0 points, (0)
Average score
Your score
Categories
Not categorized0%
Trauma0%
1
2
3
4
5
6
7
8
9
10
Answered
Review
Question 1 of 10
1. Question
A 25-year-old football player is brought in to the emergency department with neck pain and paresthesias after tackling another player head-on. Physical exam is significant for cervical spine tenderness to palpation. CT is not available, so spinal precautions are maintained and plain films are obtained. What type of fracture pattern is seen in the associated xray?
Correct
This xray shows a Jefferson fracture, or a C1 burst fracture. These fractures occur with axial loading with vertical compression, an injury pattern that is common when a football player spears another player. When looking at the odontoid view of a c-spine film, the lateral masses of C1 shoulder align with the lateral parts of the C2 vertebral body. A Hangman’s fracture, or bilateral C2 pedicle fracture is best seen on a lateral view and shows C2 displaced anteriorly on C3. Odontoid fractures involve the odontoid process and are graded type I through type III depending on position. Type II and III are both unstable fractures. Bilateral facet dislocation fractures are seen on lateral xrays of the c-spine and is characterized by anterior displacement greater than 50% diameter of vertebral body.
Incorrect
This xray shows a Jefferson fracture, or a C1 burst fracture. These fractures occur with axial loading with vertical compression, an injury pattern that is common when a football player spears another player. When looking at the odontoid view of a c-spine film, the lateral masses of C1 shoulder align with the lateral parts of the C2 vertebral body. A Hangman’s fracture, or bilateral C2 pedicle fracture is best seen on a lateral view and shows C2 displaced anteriorly on C3. Odontoid fractures involve the odontoid process and are graded type I through type III depending on position. Type II and III are both unstable fractures. Bilateral facet dislocation fractures are seen on lateral xrays of the c-spine and is characterized by anterior displacement greater than 50% diameter of vertebral body.
Question 2 of 10
2. Question
A 45-year old male is brought in by EMS after being found down. Physical exam is significant for ecchymosis around the left orbit, multiple teeth avulsions, as well as C-spine tenderness to palpation. C-spine xray is shown below. Which of the following is correct regarding this patient’s injuries?
Correct
The correct answer is that this injury is associated with anterior cord syndrome. This patient has a flexion teardrop injury at C5 (note the wedge shaped fracture at the base of the vertebral body). This is generally caused by extreme flexion and axial load, classically a diving injury. There is failure of the anterior column, often middle column, and disruption of the posterior ligamentous complex and the significant potential for cord injury from retropulsion of a fragment as a result.
Incorrect
The correct answer is that this injury is associated with anterior cord syndrome. This patient has a flexion teardrop injury at C5 (note the wedge shaped fracture at the base of the vertebral body). This is generally caused by extreme flexion and axial load, classically a diving injury. There is failure of the anterior column, often middle column, and disruption of the posterior ligamentous complex and the significant potential for cord injury from retropulsion of a fragment as a result.
Question 3 of 10
3. Question
A 50-year-old skier collides with a tree. A head CT is shown below. What is the underlying pathophysiology?
Correct
The patient has an epidural hematoma. Epidural hematomas result from blunt trauma to the temporal or temporoparietal area with skull fracture and disruption of the middle meningeal artery. Blood pools in the potential space between the skull and dura mater, leading to a classic biconvex-appearing hematoma. Patients classically present with an initial loss of consciousness followed by a “lucid interval” and subsequent rapid neurologic decline as mass effect and herniation develops. Treatment is surgical evacuation of the hematoma.
Incorrect
The patient has an epidural hematoma. Epidural hematomas result from blunt trauma to the temporal or temporoparietal area with skull fracture and disruption of the middle meningeal artery. Blood pools in the potential space between the skull and dura mater, leading to a classic biconvex-appearing hematoma. Patients classically present with an initial loss of consciousness followed by a “lucid interval” and subsequent rapid neurologic decline as mass effect and herniation develops. Treatment is surgical evacuation of the hematoma.
Question 4 of 10
4. Question
A 13-year-old boy with no past medical history presents with a headache 3 days after a closed head injury. The patient states that he stood up from kneeling and hit the top of his head on a wood cabinet. There was no loss of consciousness or seizure activity. In addition to the headache, he complains of difficulty concentrating at work and dizziness. His physical examination is unremarkable. What management is indicated?
Correct
The patient presents with minor head trauma and complaints consistent with a concussion and should have neurology follow up arranged. A concussion is a minor traumatic brain injury (TBI) that is often seen in MVCs and collision sports (football, hockey). It is typically caused by a rotational injury or an acceleration-deceleration injury. Patients will present with a number of non-specific symptoms including headaches, dizziness, confusion, amnesia, difficulty concentrating, and blurry vision but do not have focal neurologic findings. Despite the absence of severe intracranial injury, patients can have chronic and debilitating symptoms from concussions. Neurology referral is recommended, as patients should have functional testing and tracking of their symptoms for resolution. It is vital to council patients to avoid contact sports or activities that increased the risk of recurrent injury as these patients are at risk for more severe injury with second impact.
Incorrect
The patient presents with minor head trauma and complaints consistent with a concussion and should have neurology follow up arranged. A concussion is a minor traumatic brain injury (TBI) that is often seen in MVCs and collision sports (football, hockey). It is typically caused by a rotational injury or an acceleration-deceleration injury. Patients will present with a number of non-specific symptoms including headaches, dizziness, confusion, amnesia, difficulty concentrating, and blurry vision but do not have focal neurologic findings. Despite the absence of severe intracranial injury, patients can have chronic and debilitating symptoms from concussions. Neurology referral is recommended, as patients should have functional testing and tracking of their symptoms for resolution. It is vital to council patients to avoid contact sports or activities that increased the risk of recurrent injury as these patients are at risk for more severe injury with second impact.
Question 5 of 10
5. Question
A 23-year-old male is involved in a high speed motor vehicle accident. He has multiple injuries, including a fracture at the junction of the odontoid and body of C2. What type of fracture does this patient have?
Correct
Odontoid (dens) fractures are classified as Type I, II and III. Type I fractures are stable avulsion injuries to the tip of the odontoid. This patient has a Type II fracture, which is a fracture at the junction of the odontoid and body of C2. A Type III fracture is a fracture at the base of the the dens. Both Type II and III fractures are unstable because of ligamentous attachments. A bilateral C2 pedicle fracture is also known as a Hangman’s fracture and is seen on lateral xray as C2 being displaced anteriorly on C3. A teardrop fracture is an avulsion fracture of the anteroinferior portion of the vertebral body. Both of the former fractures patterns are unstable c-spine injuries.
Incorrect
Odontoid (dens) fractures are classified as Type I, II and III. Type I fractures are stable avulsion injuries to the tip of the odontoid. This patient has a Type II fracture, which is a fracture at the junction of the odontoid and body of C2. A Type III fracture is a fracture at the base of the the dens. Both Type II and III fractures are unstable because of ligamentous attachments. A bilateral C2 pedicle fracture is also known as a Hangman’s fracture and is seen on lateral xray as C2 being displaced anteriorly on C3. A teardrop fracture is an avulsion fracture of the anteroinferior portion of the vertebral body. Both of the former fractures patterns are unstable c-spine injuries.
Question 6 of 10
6. Question
A 26-year-old man presents by ambulance after being stabbed in the neck. Paramedics established large-bore intravenous access in the patient’s right upper extremity in the field, and fluids are infusing. Vital signs include BP 118/72, P 115, R 24; SpO2 is 99% on 6 L oxygen via nasal cannula. On examination, the patient is awake and alert. The wound is on his left anterior neck between the cricoid cartilage and the angle of the mandible. There is active bleeding from the site and an underlying expanding hematoma. The patient says his throat is tight and that he cannot swallow, and his girlfriend says his voice sounds strange.
In addition to applying direct pressure to the wound, what is the best next step in management?
Correct
This patient has sustained a Zone II neck injury. He is demonstrating signs of impending airway obstruction with an expanding anterior neck hematoma, dysphagia, and dysphonia. The primary focus in the care of this patient and any other patient presenting following a traumatic injury should be the ABCs. This particular patient should be intubated right away: he is at risk for rapid deterioration and respiratory failure. Indications for establishing a definitive airway in the setting of penetrating neck trauma include respiratory distress, altered mental status, bloody secretions in the oropharynx, subcutaneous emphysema, expanding hematoma, and tracheal shift. Other signs of potential airway compromise include dysphagia and dysphonia. Orotracheal intubation is preferred in penetrating neck injury, but backup airway techniques should be available. If quickly available, awake fiberoptic intubation may be considered.
The patient’s presentation is critical. As soon as the airway is established, he should be transported immediately to the OR. Performing angiography would inappropriately delay management. All symptomatic patients with penetrating injuries to Zone II require surgical exploration.
Again, sending the patient for a CT scan would delay appropriate management. Additionally, CT is more appropriate in the evaluation of blunt trauma.
Cricothyrotomy should be avoided when an anterior neck hematoma is present. If the patient were more stable on arrival, intubation could be delayed until the patient reached the OR.
Incorrect
This patient has sustained a Zone II neck injury. He is demonstrating signs of impending airway obstruction with an expanding anterior neck hematoma, dysphagia, and dysphonia. The primary focus in the care of this patient and any other patient presenting following a traumatic injury should be the ABCs. This particular patient should be intubated right away: he is at risk for rapid deterioration and respiratory failure. Indications for establishing a definitive airway in the setting of penetrating neck trauma include respiratory distress, altered mental status, bloody secretions in the oropharynx, subcutaneous emphysema, expanding hematoma, and tracheal shift. Other signs of potential airway compromise include dysphagia and dysphonia. Orotracheal intubation is preferred in penetrating neck injury, but backup airway techniques should be available. If quickly available, awake fiberoptic intubation may be considered.
The patient’s presentation is critical. As soon as the airway is established, he should be transported immediately to the OR. Performing angiography would inappropriately delay management. All symptomatic patients with penetrating injuries to Zone II require surgical exploration.
Again, sending the patient for a CT scan would delay appropriate management. Additionally, CT is more appropriate in the evaluation of blunt trauma.
Cricothyrotomy should be avoided when an anterior neck hematoma is present. If the patient were more stable on arrival, intubation could be delayed until the patient reached the OR.
Question 7 of 10
7. Question
Which of the following interventions used to reduce intracranial pressure in head-injured infants and children has been shown to be the most effective for most patients?
Correct
The treatment and amelioration of increased intracranial pressure (ICP) is one of the most important steps in the management of the multiple trauma patient. Mannitol is very effective, with a slightly delayed effect on the oncotic forces at work within the cranial cavity. Indicated in patients with impending herniation or evidence of increased ICP from trauma, mannitol is given as a bolus to lower ICP via osmotic diuresis and increase plasma expansion. A continuous mannitol drip can increase the chance of hypovolemia and is thus not recommended.
Hyperventilation after intubation has been used as a rapid intervention to lower ICP. It can be used for impending herniation or as a lifesaving therapeutic measure. But more recent studies indicate that patients with traumatic brain injury have a worse prognosis if hyperventilation is used in any prolonged manner, and that it should be avoided especially in the first 24 hours.
Neuromuscular paralysis is part of the process of rapid sequence intubation, but it does not in itself contribute to reduction of ICP. Successful intubation to manually control the patient’s respiratory rate and ventilation volume is made more successful with the use of neuromuscular blockers. Complete paralysis allows mechanical hyperventilation while the patient is on the ventilator.
The value of steroids in the care of patients with increased ICP from other processes, including cancer, is debatable. Several large studies have shown that steroids are not effective in treating patients with traumatic brain injury.
Incorrect
The treatment and amelioration of increased intracranial pressure (ICP) is one of the most important steps in the management of the multiple trauma patient. Mannitol is very effective, with a slightly delayed effect on the oncotic forces at work within the cranial cavity. Indicated in patients with impending herniation or evidence of increased ICP from trauma, mannitol is given as a bolus to lower ICP via osmotic diuresis and increase plasma expansion. A continuous mannitol drip can increase the chance of hypovolemia and is thus not recommended.
Hyperventilation after intubation has been used as a rapid intervention to lower ICP. It can be used for impending herniation or as a lifesaving therapeutic measure. But more recent studies indicate that patients with traumatic brain injury have a worse prognosis if hyperventilation is used in any prolonged manner, and that it should be avoided especially in the first 24 hours.
Neuromuscular paralysis is part of the process of rapid sequence intubation, but it does not in itself contribute to reduction of ICP. Successful intubation to manually control the patient’s respiratory rate and ventilation volume is made more successful with the use of neuromuscular blockers. Complete paralysis allows mechanical hyperventilation while the patient is on the ventilator.
The value of steroids in the care of patients with increased ICP from other processes, including cancer, is debatable. Several large studies have shown that steroids are not effective in treating patients with traumatic brain injury.
Question 8 of 10
8. Question
A 26-year old male is brought in to the Emergency Department after falling from his bicycle. The patient was not helmeted and is endorsing significant headache. Physical exam reveals a large parietal hematoma as well as blood-tinged liquid from the left ear. The patient has no other injuries. In addition to imaging to exclude any other injuries, which of the following is the most appropriate management of this patient’s condition?
Correct
A. Admission to the hospital
This patient is presenting with a basilar skull fracture, evidenced by the CSF fluid coming from his ear. Recommendations are that all of these patients should be admitted to the hospital for monitoring. A recent cochrane review found no evidence that routine antibiotics were indicated for CSF leaks in these patients however. Source: Ratilal BO, Costa J, Pappamikail L, Sampaio C, Antibiotic prophylaxis for preventing meningitis in patients with basilar skull fractures. Cochrane Database Syst Rev. 2015;4:CD004884. PMID: 25918919
B. Admission and systemic antibiotic therapy
While this patient should be admitted for his basilar skull fracture, antibiotics are not indicated. A recent cochrane review found no evidence that routine antibiotics were indicated for CSF leaks in these patients however. Source: Ratilal BO, Costa J, Pappamikail L, Sampaio C, Antibiotic prophylaxis for preventing meningitis in patients with basilar skull fractures. Cochrane Database Syst Rev. 2015;4:CD004884. PMID: 25918919
C. Admission and systemic steroids
While this patient should be admitted for his basilar skull fracture, steroids are not indicated
D. Discharge with oral antibiotics
This has a basilar skull fracture and CSF leak, and should be admitted for monitoring.
Incorrect
A. Admission to the hospital
This patient is presenting with a basilar skull fracture, evidenced by the CSF fluid coming from his ear. Recommendations are that all of these patients should be admitted to the hospital for monitoring. A recent cochrane review found no evidence that routine antibiotics were indicated for CSF leaks in these patients however. Source: Ratilal BO, Costa J, Pappamikail L, Sampaio C, Antibiotic prophylaxis for preventing meningitis in patients with basilar skull fractures. Cochrane Database Syst Rev. 2015;4:CD004884. PMID: 25918919
B. Admission and systemic antibiotic therapy
While this patient should be admitted for his basilar skull fracture, antibiotics are not indicated. A recent cochrane review found no evidence that routine antibiotics were indicated for CSF leaks in these patients however. Source: Ratilal BO, Costa J, Pappamikail L, Sampaio C, Antibiotic prophylaxis for preventing meningitis in patients with basilar skull fractures. Cochrane Database Syst Rev. 2015;4:CD004884. PMID: 25918919
C. Admission and systemic steroids
While this patient should be admitted for his basilar skull fracture, steroids are not indicated
D. Discharge with oral antibiotics
This has a basilar skull fracture and CSF leak, and should be admitted for monitoring.
Question 9 of 10
9. Question
27-year-old man is involved in a motor vehicle collision in which he was the unrestrained driver. He presents to the Emergency Department with significant facial trauma. A CT of his face shows a Le Fort type III fracture. Which of the following structures is affected?
Correct
Complex facial fractures may be classified into the Le Fort system of fracture patterns. Three types exist. Type I Le Fort fractures are transverse fractures through the base of the maxilla and often involve the roots of the teeth. It may be unilateral and bilateral. The upper teeth may be grasped and rocked independent of the midface, indicating discontinuity with the maxilla. Type II Le Fort fractures are often bilateral and are characterized by pyramidal fractures that extend through the body of maxilla, orbital floor, nasal bones, hard palate, and lacrimal bones. The nasal unit and maxilla may be grasped and move as a unit relative to the orbital complexes and lower face. Type III Le Fort fractures are considered complete craniofacial disjunction as the fracture extends from the nasal bridge traveling posteriorly along the ethmoid bone, which comprises the medial orbital wall, and continues through the inferior and lateral orbital walls and frontozygomatic suture. The sphenoid bone is often involved with type III fractures, and cerebrospinal fluid (CSF) leak may occur. CSF may also leak from involvement of the cribriform plate of the ethmoid bones. Le Fort fractures are best seen with computed tomography. Depending on the extent of the injury, Le Fort fractures often require management of a multidisciplinary team and may necessitate consultation to otolaryngology, plastic and reconstructive surgery, dentistry and neurosurgery if a CSF leak is present.
A type II Le Fort fracture involves the central maxilla (A), nasal bridge, lacrimal bones, hard palate (C), and orbital floor. It often occurs bilaterally and in a pyramidal pattern. Fractures of the mandibular condyle (D) are not included in the Le Fort classification of midface fractures.
Incorrect
Complex facial fractures may be classified into the Le Fort system of fracture patterns. Three types exist. Type I Le Fort fractures are transverse fractures through the base of the maxilla and often involve the roots of the teeth. It may be unilateral and bilateral. The upper teeth may be grasped and rocked independent of the midface, indicating discontinuity with the maxilla. Type II Le Fort fractures are often bilateral and are characterized by pyramidal fractures that extend through the body of maxilla, orbital floor, nasal bones, hard palate, and lacrimal bones. The nasal unit and maxilla may be grasped and move as a unit relative to the orbital complexes and lower face. Type III Le Fort fractures are considered complete craniofacial disjunction as the fracture extends from the nasal bridge traveling posteriorly along the ethmoid bone, which comprises the medial orbital wall, and continues through the inferior and lateral orbital walls and frontozygomatic suture. The sphenoid bone is often involved with type III fractures, and cerebrospinal fluid (CSF) leak may occur. CSF may also leak from involvement of the cribriform plate of the ethmoid bones. Le Fort fractures are best seen with computed tomography. Depending on the extent of the injury, Le Fort fractures often require management of a multidisciplinary team and may necessitate consultation to otolaryngology, plastic and reconstructive surgery, dentistry and neurosurgery if a CSF leak is present.
A type II Le Fort fracture involves the central maxilla (A), nasal bridge, lacrimal bones, hard palate (C), and orbital floor. It often occurs bilaterally and in a pyramidal pattern. Fractures of the mandibular condyle (D) are not included in the Le Fort classification of midface fractures.
Question 10 of 10
10. Question
Bedside ultrasonography performed by an emergency physician may accurately diagnose all of the following conditions EXCEPT:
Correct
Bedside intraocular ultrasound performed by an emergency physician is a very useful tool for rapidly diagnosing many intraocular conditions. Dislocated lens, foreign body, and detached retina can be identified by ultrasound. It has not been demonstrated as useful in the diagnosis of orbital compartment syndrome caused by a retrobulbar hemorrhage.
Incorrect
Bedside intraocular ultrasound performed by an emergency physician is a very useful tool for rapidly diagnosing many intraocular conditions. Dislocated lens, foreign body, and detached retina can be identified by ultrasound. It has not been demonstrated as useful in the diagnosis of orbital compartment syndrome caused by a retrobulbar hemorrhage.
First, enjoy a nice holiday break from conference. Instead of conference, enjoy some time on Thursday morning with family, or disimpacting a patient in the TCU.
After the holiday break, we will begin a months worth of trauma, beginning with Head, Neck, Maxillofacial. There is a lot of content coming up, and it is all high yield for both boards and shifts!
We will begin with a quiz review (this is the sesquicentennial quiz for our program! Quiz #150!), followed by F/U rounds with Dr. Buscarino, followed by FLIP stations by Drs. Vaizer and Sykes. We will then feast while Journal club stuff, hosted by Dr. McElroy and Dr. Olsen.
You have already completed the quiz before. Hence you can not start it again.
Quiz is loading...
You must sign in or sign up to start the quiz.
You have to finish following quiz, to start this quiz:
Results
0 of 10 questions answered correctly
Your time:
Time has elapsed
You have reached 0 of 0 points, (0)
Average score
Your score
Categories
Environmental0%
1
2
3
4
5
6
7
8
9
10
Answered
Review
Question 1 of 10
1. Question
A 41-year-old woman presents to the emergency tent with generalized weakness, dizziness, and nausea after running an 18-mile race. The local temperature is 90℉ with relative humidity of 80%. As you continue your evaluation, what physical exam finding would make you concerned that this patient is developing a more severe illness requiring more emergent management?
Correct
Heat emergencies represent a continuum of disorders that range in severity from heat cramps to heat exhaustion to life-threatening heat stroke. In most circumstances, heat emergencies can be avoided through common sense, public education, and prevention. People most at risk for heat injuries include athletes, military personnel, and laborers who work outdoors in the heat. Classic heat injury, as is the case with this patient, usually occurs during periods of high environmental heat stress. Physical exertion is not required if environmental temperatures and humidity create a situation where heat gain overwhelms native heat losses. Exertional heat injury usually affects individuals who are participating in athletic events in conditions of high heat stress. Clinical features are what determine the severity of heat illness. This patient is presenting with symptoms characteristic of heat exhaustion. Other symptoms can include muscle cramps, headache, and vomiting. On physical examination of patients with heat exhaustion, the temperature may be normal or elevated. However, patients with heat exhaustion will not have mental status changes. Mental status changes are a sign of heat stroke which can be life-threatening.
Dry skin, or anhidrosis (A), can classically be found in patients with heat stroke but this is not a requirement for the diagnosis since patients with severe heat illness can also present with diaphoresis. Heat exhaustion is characterized by the inability to maintain adequate cardiac output due to strenuous exercise and environmental heat stress. Acute dehydration may be present. Athletes with heat exhaustion have difficulty continuing with exercise. Other signs of heat exhaustion include hypotension (B), syncope, and tachycardia (D). The presence of these factors does not necessarily portend severe complications from the illness. Severe heat illness, heat stroke, presents with neurological symptoms such as hallucinations, seizure, mental status changes, or coma.
Incorrect
Heat emergencies represent a continuum of disorders that range in severity from heat cramps to heat exhaustion to life-threatening heat stroke. In most circumstances, heat emergencies can be avoided through common sense, public education, and prevention. People most at risk for heat injuries include athletes, military personnel, and laborers who work outdoors in the heat. Classic heat injury, as is the case with this patient, usually occurs during periods of high environmental heat stress. Physical exertion is not required if environmental temperatures and humidity create a situation where heat gain overwhelms native heat losses. Exertional heat injury usually affects individuals who are participating in athletic events in conditions of high heat stress. Clinical features are what determine the severity of heat illness. This patient is presenting with symptoms characteristic of heat exhaustion. Other symptoms can include muscle cramps, headache, and vomiting. On physical examination of patients with heat exhaustion, the temperature may be normal or elevated. However, patients with heat exhaustion will not have mental status changes. Mental status changes are a sign of heat stroke which can be life-threatening.
Dry skin, or anhidrosis (A), can classically be found in patients with heat stroke but this is not a requirement for the diagnosis since patients with severe heat illness can also present with diaphoresis. Heat exhaustion is characterized by the inability to maintain adequate cardiac output due to strenuous exercise and environmental heat stress. Acute dehydration may be present. Athletes with heat exhaustion have difficulty continuing with exercise. Other signs of heat exhaustion include hypotension (B), syncope, and tachycardia (D). The presence of these factors does not necessarily portend severe complications from the illness. Severe heat illness, heat stroke, presents with neurological symptoms such as hallucinations, seizure, mental status changes, or coma.
Question 2 of 10
2. Question
A 46-year-old man presents to the emergency department with leg and abdominal cramping that started one hour ago. The patient was working in his yard trimming trees when he developed cramping in his right leg. He does not recall falling or twisting his leg in any fashion. The cramping now affects both of his legs and he has developed abdominal pain, too. He vomited one time. He was well earlier in the day. Vital signs show HR 118, BP 178/100, RR 16, and T 37.6℃. On exam, you find a man in obvious distress due to cramping abdominal pain and a tense, rigid, diffusely tender abdomen with diffuse tenderness in both of his legs. What is the most likely environmental cause of this patient’s symptoms?
Correct
The class Arachnida contains the largest number of venomous species known, but most are not harmful to humans. Humans fear spiders and scorpions, but of this class of species ticks probably cause the greatest morbidity. One of the more venomous spiders known is the black widow spider or Latrodectus species. They are found throughout the United States, except Alaska. Symptoms of a black widow spider bite include a pinprick sensation followed by local redness and swelling, but sometimes the bite is not felt. Over the next hour, patients develop dull cramping in the area of the bite that eventually spreads to the entire body. Pain is centered in the chest in patients who suffer an upper extremity bite, and pain is centered in the abdomen in patients who suffer a lower extremity bite. The abdomen may become boardlike and can mimic pancreatitis, appendicitis, or a perforated peptic ulcer. Signs and symptoms usually abate within a few hours and definitely resolve in two to three days. Management is supportive with benzodiazepine treatment for muscle spasms.
Symptoms of a brown recluse spider (B) bite include local burning pain at the site of the bite. After a few hours, a bleb forms in the center of the bite with a surrounding erythematous ring. The area can look like a target sign or a bull’s eye. Over the next few days, the bleb darkens and begins to necrose with eventual involvement of the underlying subcutaneous fat along with the skin. Systemic symptoms such as fever, malaise, vomiting, and rash can develop and lead to shock in severe cases, although fatalities are rare. This patient does not have a bite characteristic of a brown recluse spider. A scorpion (C) sting is characterized by immediate pain at the site of the sting. Over some time, patients can develop numbness and weakness locally and will have heightened sensitivity in the sting area. Systemic symptoms such as restlessness, vomiting, muscle spasms, nystagmus, and myoclonus can develop in severe cases but are more rare. Scorpion sting symptoms can mimic those found in patients with black widow spider bites but are much less common. Most tarantulas (D) are nontoxic. They are unusual in that their abdominal hairs can be thrown and embedded in human skin, leading to allergic reactions. There is not a toxic bite. The patient in this question is not presenting with allergic reaction symptoms.
Incorrect
The class Arachnida contains the largest number of venomous species known, but most are not harmful to humans. Humans fear spiders and scorpions, but of this class of species ticks probably cause the greatest morbidity. One of the more venomous spiders known is the black widow spider or Latrodectus species. They are found throughout the United States, except Alaska. Symptoms of a black widow spider bite include a pinprick sensation followed by local redness and swelling, but sometimes the bite is not felt. Over the next hour, patients develop dull cramping in the area of the bite that eventually spreads to the entire body. Pain is centered in the chest in patients who suffer an upper extremity bite, and pain is centered in the abdomen in patients who suffer a lower extremity bite. The abdomen may become boardlike and can mimic pancreatitis, appendicitis, or a perforated peptic ulcer. Signs and symptoms usually abate within a few hours and definitely resolve in two to three days. Management is supportive with benzodiazepine treatment for muscle spasms.
Symptoms of a brown recluse spider (B) bite include local burning pain at the site of the bite. After a few hours, a bleb forms in the center of the bite with a surrounding erythematous ring. The area can look like a target sign or a bull’s eye. Over the next few days, the bleb darkens and begins to necrose with eventual involvement of the underlying subcutaneous fat along with the skin. Systemic symptoms such as fever, malaise, vomiting, and rash can develop and lead to shock in severe cases, although fatalities are rare. This patient does not have a bite characteristic of a brown recluse spider. A scorpion (C) sting is characterized by immediate pain at the site of the sting. Over some time, patients can develop numbness and weakness locally and will have heightened sensitivity in the sting area. Systemic symptoms such as restlessness, vomiting, muscle spasms, nystagmus, and myoclonus can develop in severe cases but are more rare. Scorpion sting symptoms can mimic those found in patients with black widow spider bites but are much less common. Most tarantulas (D) are nontoxic. They are unusual in that their abdominal hairs can be thrown and embedded in human skin, leading to allergic reactions. There is not a toxic bite. The patient in this question is not presenting with allergic reaction symptoms.
Question 3 of 10
3. Question
A 26-year-woman is brought to the emergency department by her diving partner. Per the partner, the patient developed cough, shortness of breath, ataxia, and vertigo one hour after a long, deep dive. The patient is in moderate respiratory distress, and complains of chest pain and severe dizziness. Which of the following is the most likely diagnosis?
Correct
Decompression sickness occurs due to the liberation of gas bubbles associated with a decrease in ambient pressure. This can occur in divers breathing compressed air as well as high altitude pilots or astronauts. The gas bubbles likely obstruct blood flow, leading to direct tissue ischemia. They also activate a variety of inflammatory processes, leading to thrombosis and capillary leaking. Symptoms of decompression sickness typically occur minutes to hours after surfacing. Decompression sickness can include pain syndromes involving the joints (“the bends”) as well as more serious manifestations including pulmonary (“the chokes”), cardiovascular, and neurologic symptoms (“the staggers”). Treatment of decompression sickness is similar to the treatment of an arterial air embolism and includes administration of oxygen, increasing tissue perfusion with intravenous fluids, and rapid decompression with hyperbaric oxygen.
Arterial air embolism (A) is an example of a barotrauma of ascent and results from entry of air into the arterial system with resultant embolization. The brain is the most commonly affected organ. Arterial air embolism should be suspected in any diver with loss of consciousness upon surfacing. Inner ear barotrauma (C) is an example of barotrauma of rapid descent or ascent and results from rupture of the round window or tearing of the vestibular membrane due to a forceful Valsalva maneuver done during descent to clear the ears. Symptoms include vertigo (as seen in the patient above) but this condition is easily differentiated from decompression sickness due to the onset of symptoms during descent rather than after surfacing. Nitrogen narcosis (D) occurs with deep dives and includes symptoms such as loss of fine motor skills and higher-order brain functions, resulting in bizarre and dangerous behavior. Presentation is similar to alcohol intoxication.
Incorrect
Decompression sickness occurs due to the liberation of gas bubbles associated with a decrease in ambient pressure. This can occur in divers breathing compressed air as well as high altitude pilots or astronauts. The gas bubbles likely obstruct blood flow, leading to direct tissue ischemia. They also activate a variety of inflammatory processes, leading to thrombosis and capillary leaking. Symptoms of decompression sickness typically occur minutes to hours after surfacing. Decompression sickness can include pain syndromes involving the joints (“the bends”) as well as more serious manifestations including pulmonary (“the chokes”), cardiovascular, and neurologic symptoms (“the staggers”). Treatment of decompression sickness is similar to the treatment of an arterial air embolism and includes administration of oxygen, increasing tissue perfusion with intravenous fluids, and rapid decompression with hyperbaric oxygen.
Arterial air embolism (A) is an example of a barotrauma of ascent and results from entry of air into the arterial system with resultant embolization. The brain is the most commonly affected organ. Arterial air embolism should be suspected in any diver with loss of consciousness upon surfacing. Inner ear barotrauma (C) is an example of barotrauma of rapid descent or ascent and results from rupture of the round window or tearing of the vestibular membrane due to a forceful Valsalva maneuver done during descent to clear the ears. Symptoms include vertigo (as seen in the patient above) but this condition is easily differentiated from decompression sickness due to the onset of symptoms during descent rather than after surfacing. Nitrogen narcosis (D) occurs with deep dives and includes symptoms such as loss of fine motor skills and higher-order brain functions, resulting in bizarre and dangerous behavior. Presentation is similar to alcohol intoxication.
Question 4 of 10
4. Question
The most common arrhythmia in hypothermia is atrial fibrillation. How is this best managed?
Correct
Atrial fibrillation in hypothermia tends to be relatively benign and resolves spontaneously with rewarming. Persistent atrial fibrillation after complete rewarming is unlikely to be due to hypothermia and may suggest an underlying medical pathology. Cardioversion increases the risk of ventricular fibrillation and is contraindicated unless the patient is unstable. Anticoagulation for temporary atrial fibrillation is not indicated and could worsen the coagulopathy already present in hypothermia.
Incorrect
Atrial fibrillation in hypothermia tends to be relatively benign and resolves spontaneously with rewarming. Persistent atrial fibrillation after complete rewarming is unlikely to be due to hypothermia and may suggest an underlying medical pathology. Cardioversion increases the risk of ventricular fibrillation and is contraindicated unless the patient is unstable. Anticoagulation for temporary atrial fibrillation is not indicated and could worsen the coagulopathy already present in hypothermia.
Question 5 of 10
5. Question
A 20-year-old man presents with arm pain after an electrical injury. The patient was working when he touched a “live wire” and was thrown backwards. He had no loss of consciousness and only complains of pain in his arms. Vital signs are unremarkable. A urinalysis shows 3+ blood with 5-10 RBCs. What management is indicated?
Correct
This patient presents with rhabdomyolysis after an electrical injury and should be treated with aggressive intravenous hydration. Electrical injuries are common and are the cause of about 5% of burn unit admissions and 6% of occupational fatalities. The extent of the injury is related to the type of circuit (alternating current (AC) versus direct current (DC)), the resistance, amperage, duration of contact and voltage. The initial exposure to electricity can cause severe burns spreading from the point of impact. Additionally, the energy from the shock is transmitted into the body and can cause significant muscle breakdown. Breakdown is increased if the patient experiences tetany or prolonged exposure to the electrical source. The classic finding of rhabdomyolysis is a urine dip that is positive for blood with only scant or no RBCs on microscopy. The urine dipstick falsely reports myoglobin as hemoglobin in this case. A serum creatinine kinase level should be obtained. Initial treatment is with aggressive fluid resuscitation to maintain urinary output as myoglobin accumulation can lead to renal failure. Electrolyte abnormalities (specifically hyperkalemia from cellular breakdown) should be closely monitored and treated as well.
A CT scan of the abdomen (A) is not necessary to look for a bleeding source. Intravenous antibiotics (B) are not necessary as this urinalysis does not indicate the presence of infection. Packed red blood cells (D) are not needed at this time as the patient does not exhibit signs of bleeding.
Incorrect
This patient presents with rhabdomyolysis after an electrical injury and should be treated with aggressive intravenous hydration. Electrical injuries are common and are the cause of about 5% of burn unit admissions and 6% of occupational fatalities. The extent of the injury is related to the type of circuit (alternating current (AC) versus direct current (DC)), the resistance, amperage, duration of contact and voltage. The initial exposure to electricity can cause severe burns spreading from the point of impact. Additionally, the energy from the shock is transmitted into the body and can cause significant muscle breakdown. Breakdown is increased if the patient experiences tetany or prolonged exposure to the electrical source. The classic finding of rhabdomyolysis is a urine dip that is positive for blood with only scant or no RBCs on microscopy. The urine dipstick falsely reports myoglobin as hemoglobin in this case. A serum creatinine kinase level should be obtained. Initial treatment is with aggressive fluid resuscitation to maintain urinary output as myoglobin accumulation can lead to renal failure. Electrolyte abnormalities (specifically hyperkalemia from cellular breakdown) should be closely monitored and treated as well.
A CT scan of the abdomen (A) is not necessary to look for a bleeding source. Intravenous antibiotics (B) are not necessary as this urinalysis does not indicate the presence of infection. Packed red blood cells (D) are not needed at this time as the patient does not exhibit signs of bleeding.
Question 6 of 10
6. Question
A 34-year-old woman is backpacking in Utah and begins to experience a headache. Which of the following clinical features is an early sign that is most suggestive of high-altitude cerebral edema?
Correct
The partial pressure of oxygen decreases as barometric pressure decreases at altitude. At 8,000 feet or greater above sea level, the environment becomes hypoxic and high-altitude syndromes can occur due to hypoxia. These include acute mountain sickness (AMS), high-altitude pulmonary edema, high-altitude cerebral edema, retinopathy, and certain neurologic syndromes. High-altitude cerebral edema (HACE) is the most severe form of acute mountain sickness. At altitude, cerebral vasodilation occurs and leads to increased blood flow and blood volume. This is due to a leaky blood-brain barrier either from lack of autonomic autoregulation or increased vascular permeability, or a combination of the two. HACE is characterized by progressive global cerebral dysfunction and causes confusion, altered mental status, and ataxia. Ataxia is a cardinal and early sign of HACE, and all patients with acute mountain sickness should be monitored for ataxia. Other features, such as headache, nausea, or vomiting, may be absent. Retinal hemorrhages are also common, and cranial nerve palsies of the third and sixth cranial nerves may occur due to increased intracranial pressure. Concomitant pulmonary edema is common. Death from HACE is due to brainstem herniation, so early identification and management are critical. The highest priority is descent. If this is not possible, temporizing measures (though often unavailable) include supplemental oxygen, corticosteroids, and use of a portable hyperbaric oxygen chamber. Computed tomography of the head may demonstrate white matter hyperattenuation more than gray matter with effacement of the sulci and flattening of the gyri.
Incorrect
The partial pressure of oxygen decreases as barometric pressure decreases at altitude. At 8,000 feet or greater above sea level, the environment becomes hypoxic and high-altitude syndromes can occur due to hypoxia. These include acute mountain sickness (AMS), high-altitude pulmonary edema, high-altitude cerebral edema, retinopathy, and certain neurologic syndromes. High-altitude cerebral edema (HACE) is the most severe form of acute mountain sickness. At altitude, cerebral vasodilation occurs and leads to increased blood flow and blood volume. This is due to a leaky blood-brain barrier either from lack of autonomic autoregulation or increased vascular permeability, or a combination of the two. HACE is characterized by progressive global cerebral dysfunction and causes confusion, altered mental status, and ataxia. Ataxia is a cardinal and early sign of HACE, and all patients with acute mountain sickness should be monitored for ataxia. Other features, such as headache, nausea, or vomiting, may be absent. Retinal hemorrhages are also common, and cranial nerve palsies of the third and sixth cranial nerves may occur due to increased intracranial pressure. Concomitant pulmonary edema is common. Death from HACE is due to brainstem herniation, so early identification and management are critical. The highest priority is descent. If this is not possible, temporizing measures (though often unavailable) include supplemental oxygen, corticosteroids, and use of a portable hyperbaric oxygen chamber. Computed tomography of the head may demonstrate white matter hyperattenuation more than gray matter with effacement of the sulci and flattening of the gyri.
Question 7 of 10
7. Question
A 45-year-old man is brought into an emergency department in Miami by emergency medical services with right arm weakness, right facial droop, and difficulty finding words. His wife said that she had just taken the patient on his first SCUBA dive when he suddenly developed these symptoms as they were ascending to the surface. He had to be pulled from the water by the Divemaster. On examination, you note crepitus when you palpate the chest and neck. Which of the following is the most likely diagnosis?
Correct
This patient presents with signs and symptoms consistent with a left middle cerebral artery stroke as well as subcutaneous emphysema that developed during ascent from diving. Any focal neurologic deficit that results during ascent or immediately thereafter should be considered a cerebral arterial air embolism until proven otherwise. Rapid uncontrolled ascent against a closed glottis causes the volume of gas in the lungs to expand according to Boyle’s law (pressure and volume of a gas are inversely related at a constant temperature). As this gas expands, it can lead to pulmonary barotrauma, including pneumothorax, pneumomediastinum, (as indicated by this patient’s subcutaneous emphysema on exam), and alveolar rupture. Alveolar rupture leads to gas leakage into the pulmonary veins, which can then move into the systemic circulation and lodge in distal arterioles as arterial gas emboli. Cerebral gas embolism presents with typical stroke symptoms and is one of the most serious complications of pulmonary barotrauma. Air embolization can be prevented by exhaling during ascent, which prevents the rapid expansion of volume within the lungs as pressure decreases. Treatment includes fluid resuscitation and rapid recompression with 100% hyperbaric oxygen. Hyperbaric oxygen works by decreasing the size of the air embolism as well as easing washout of other gases by increasing the diffusion gradient for nitrogen from the air embolism to plasma.
Barotitis (B), or ear barotrauma, is the most common diving disorder and results from unequalized increasing pressure on the tympanic membrane on descent. Symptoms include unilateral ear pain and fullness, as well as vertigo and disorientation from rupture of the tympanic membrane and uneven caloric stimulation of the middle ear. Treatment is conservative. Decompression syndrome (C), or “the bends,” usually occurs within six hours after ascent. It results from the expansion of inert gas that dissolved in tissues during descent. As the pressure of these inert gases exceeds atmospheric pressure, it can exit tissue in the form of bubbles, which often lodge in veins, leading to localized venous obstruction and inflammatory cascades. Patients either present with significant pain (type I) or ascending paresthesias or paralysis (type II). Treatment is the same as for arterial gas embolization. Nitrogen narcosis (D) results from the increased partial pressure of nitrogen in CNS tissue and therefore occurs at significant depth. Symptoms are similar to alcohol intoxication and include altered coordination and impaired judgment.
Incorrect
This patient presents with signs and symptoms consistent with a left middle cerebral artery stroke as well as subcutaneous emphysema that developed during ascent from diving. Any focal neurologic deficit that results during ascent or immediately thereafter should be considered a cerebral arterial air embolism until proven otherwise. Rapid uncontrolled ascent against a closed glottis causes the volume of gas in the lungs to expand according to Boyle’s law (pressure and volume of a gas are inversely related at a constant temperature). As this gas expands, it can lead to pulmonary barotrauma, including pneumothorax, pneumomediastinum, (as indicated by this patient’s subcutaneous emphysema on exam), and alveolar rupture. Alveolar rupture leads to gas leakage into the pulmonary veins, which can then move into the systemic circulation and lodge in distal arterioles as arterial gas emboli. Cerebral gas embolism presents with typical stroke symptoms and is one of the most serious complications of pulmonary barotrauma. Air embolization can be prevented by exhaling during ascent, which prevents the rapid expansion of volume within the lungs as pressure decreases. Treatment includes fluid resuscitation and rapid recompression with 100% hyperbaric oxygen. Hyperbaric oxygen works by decreasing the size of the air embolism as well as easing washout of other gases by increasing the diffusion gradient for nitrogen from the air embolism to plasma.
Barotitis (B), or ear barotrauma, is the most common diving disorder and results from unequalized increasing pressure on the tympanic membrane on descent. Symptoms include unilateral ear pain and fullness, as well as vertigo and disorientation from rupture of the tympanic membrane and uneven caloric stimulation of the middle ear. Treatment is conservative. Decompression syndrome (C), or “the bends,” usually occurs within six hours after ascent. It results from the expansion of inert gas that dissolved in tissues during descent. As the pressure of these inert gases exceeds atmospheric pressure, it can exit tissue in the form of bubbles, which often lodge in veins, leading to localized venous obstruction and inflammatory cascades. Patients either present with significant pain (type I) or ascending paresthesias or paralysis (type II). Treatment is the same as for arterial gas embolization. Nitrogen narcosis (D) results from the increased partial pressure of nitrogen in CNS tissue and therefore occurs at significant depth. Symptoms are similar to alcohol intoxication and include altered coordination and impaired judgment.
Question 8 of 10
8. Question
Which would you expect in a patient with the following ECG?
Correct
The ECG demonstrates the presence of J waves or Osborn waves which are seen in hypothermia. One of the first cardiac effects of hypothermia is bradycardia secondary to decreased firing of the cardiac pacemaker cells in cold temperatures. Osborn waves may appear at any temperature below 32°C. The waves are an upward deflection at the terminal portion of the QRS complex. They may represent abnormal ion flux in cold temperatures along with delayed depolarization and early repolarization of the left ventricular wall. As temperatures continue to drop, the ECG will demonstrate prolonged intervals: PR, followed by QRS and then QTc.
Both diabetic ketoacidosis (A) and digoxin toxicity (B) may lead to hyperkalemia. In diabetic ketoacidosis, hyperkalemia develops as a result of the acidic pH in the blood and the transport of hydrogen ions intracellularly in exchange for a potassium ion. Digoxin toxicity poisons the cellular Na+/K+ ATPase resulting in elevated extracellular levels of potassium. The ECG manifestations of hyperkalemia begin with peaked T waves. Multiple other findings eventually develop including a shortened QT interval, ST depression, bundle branch blocks, widened QRS, prolonged PR interval, flattened T wave and ultimately a sine wave. Hyperparathyroidism (C) may lead to hypercalcemia. In hypercalcemia, the ECG shows a shortened QT interval, flattened T waves and QRS widening at very high levels.
Incorrect
The ECG demonstrates the presence of J waves or Osborn waves which are seen in hypothermia. One of the first cardiac effects of hypothermia is bradycardia secondary to decreased firing of the cardiac pacemaker cells in cold temperatures. Osborn waves may appear at any temperature below 32°C. The waves are an upward deflection at the terminal portion of the QRS complex. They may represent abnormal ion flux in cold temperatures along with delayed depolarization and early repolarization of the left ventricular wall. As temperatures continue to drop, the ECG will demonstrate prolonged intervals: PR, followed by QRS and then QTc.
Both diabetic ketoacidosis (A) and digoxin toxicity (B) may lead to hyperkalemia. In diabetic ketoacidosis, hyperkalemia develops as a result of the acidic pH in the blood and the transport of hydrogen ions intracellularly in exchange for a potassium ion. Digoxin toxicity poisons the cellular Na+/K+ ATPase resulting in elevated extracellular levels of potassium. The ECG manifestations of hyperkalemia begin with peaked T waves. Multiple other findings eventually develop including a shortened QT interval, ST depression, bundle branch blocks, widened QRS, prolonged PR interval, flattened T wave and ultimately a sine wave. Hyperparathyroidism (C) may lead to hypercalcemia. In hypercalcemia, the ECG shows a shortened QT interval, flattened T waves and QRS widening at very high levels.
Question 9 of 10
9. Question
Which lab abnormality would be most commonly present in the setting of classic heat stroke?
Correct
Hepatic injury, as denoted by aspartate aminotransferase 2100 U/L, alanine aminotransferase 1900 U/L, is so extremely common in heat stroke that its absence should push one to reconsider the diagnosis. The hallmarks of heat stroke include severe hyperthermia (> 40.5oC) and neurologic dysfunction. Neurologic dysfunction occurs due to a progressive increase in heat stores because of the body’s decreasing ability to perfuse the distal extremities. As the heat stores increase, intracranial pressure increases with a subsequent decrease in cerebral blood flow. There are two types of heat stroke, each with their own patient population and clinical presentation.Classic heatstroke is due to exogenous heat production, while exertional heatstroke is due to endogenous heat production. Patients with classic heat stroke are commonly elderly with chronic diseases or those with limited access to shelter against the heat or oral hydration that present with symptoms usually in the setting of a prolonged, severe heat wave. Patients with exertional heat stroke are commonly young, athletes, or military personnel that present with symptoms after strenuous exercise in the heat. Classic heat stroke patients are usually anhidrotic, while those with exertional heat stroke tend to present diaphoretic. Diagnosis is primarily clinical, though laboratory testing should always be performed, as end-organ damage is common and can help further differentiate the types of heat stroke. Classic heat stroke tends to cause respiratory alkalosis, mild coagulopathies, mild creatinine kinase elevations, and elevated troponin. It is more common to see lactic acidosis, disseminated intravascular coagulation, rhabdomyolysis, and hypoglycemia in those with exertional heat stroke. With a mortality rate of 21–63%, treatment must be immediate and aggressive. Cooling is the mainstay of treatment with the primary goal to maximize evaporative cooling and prevent heat-generating shivering. Antipyretics are not useful in cooling, as they counteract hypothalamic-mediated hyperthermia and not environmental hyperthermia. Alongside cooling, immediate supportive treatment with intravenous fluids, supplemental oxygen, and reversal of coagulopathies should be started.
Acute kidney injury, as denoted by blood urea nitrogen 30 mg/dL, creatinine 1.5 mg/dL (B), and elevated troponin 0.8 ng/mL (D) can occur in classic heat stroke but are rare in exertional heat stroke. Rhabdomyolysis, as denoted by creatinine kinase 2500 U/L (C), is common in exertional heat stroke and less so in classic heat stroke. While each of these laboratory findings is possible in heat stroke, hepatic damage is still the most common finding.
Incorrect
Hepatic injury, as denoted by aspartate aminotransferase 2100 U/L, alanine aminotransferase 1900 U/L, is so extremely common in heat stroke that its absence should push one to reconsider the diagnosis. The hallmarks of heat stroke include severe hyperthermia (> 40.5oC) and neurologic dysfunction. Neurologic dysfunction occurs due to a progressive increase in heat stores because of the body’s decreasing ability to perfuse the distal extremities. As the heat stores increase, intracranial pressure increases with a subsequent decrease in cerebral blood flow. There are two types of heat stroke, each with their own patient population and clinical presentation.Classic heatstroke is due to exogenous heat production, while exertional heatstroke is due to endogenous heat production. Patients with classic heat stroke are commonly elderly with chronic diseases or those with limited access to shelter against the heat or oral hydration that present with symptoms usually in the setting of a prolonged, severe heat wave. Patients with exertional heat stroke are commonly young, athletes, or military personnel that present with symptoms after strenuous exercise in the heat. Classic heat stroke patients are usually anhidrotic, while those with exertional heat stroke tend to present diaphoretic. Diagnosis is primarily clinical, though laboratory testing should always be performed, as end-organ damage is common and can help further differentiate the types of heat stroke. Classic heat stroke tends to cause respiratory alkalosis, mild coagulopathies, mild creatinine kinase elevations, and elevated troponin. It is more common to see lactic acidosis, disseminated intravascular coagulation, rhabdomyolysis, and hypoglycemia in those with exertional heat stroke. With a mortality rate of 21–63%, treatment must be immediate and aggressive. Cooling is the mainstay of treatment with the primary goal to maximize evaporative cooling and prevent heat-generating shivering. Antipyretics are not useful in cooling, as they counteract hypothalamic-mediated hyperthermia and not environmental hyperthermia. Alongside cooling, immediate supportive treatment with intravenous fluids, supplemental oxygen, and reversal of coagulopathies should be started.
Acute kidney injury, as denoted by blood urea nitrogen 30 mg/dL, creatinine 1.5 mg/dL (B), and elevated troponin 0.8 ng/mL (D) can occur in classic heat stroke but are rare in exertional heat stroke. Rhabdomyolysis, as denoted by creatinine kinase 2500 U/L (C), is common in exertional heat stroke and less so in classic heat stroke. While each of these laboratory findings is possible in heat stroke, hepatic damage is still the most common finding.
Question 10 of 10
10. Question
Which of the following is NOT an effective treatment or prophylaxis of high altitude cerebral edema?
Correct
Incorrect
(P.S. Don’t forget your ugly sweater this week!)
Welcome back to FLIP! This standalone week ambitiously covers all things environmental – Earth, Fire, Water, Lightning, and more! We will start off with journal club with Dr. Wong and O’Neil. Followed by Follow Up rounds by Dr. Vandenberg, followed by a very special environmental FLIP with Drs. McElroy and Dr. Bajkowksi.
Yes, I know, there is a lot cover here in one week. Do what you can, hit the main chapters. We will cover bites at a later time, so skim over this for now. EMRAP also does a 1 hr review of everything bites/envenomations if thats your thing. Just pick your weaknesses and get through what you can!